Musculoskeletal/Orthopaedic/Rheumatology Flashcards

1
Q

A 28-year-old man is brought to the Emergency Department after he had an accident while driving a motorcycle and had his right ankle injured. On examination, his vital signs are stable. The right ankle joint is laterally displaced and there is a laceration over the joint. Which one of the following is the most important initial step in management?

A. Wound debridment.
B. Tetanus immunization.
C. Intravenous antibiotics.
D. Reduction of the displacement.
E. X-ray of the joint.

A

D. Reduction of the displacement

Open fractures and/or dislocations are defined as the bone and/or joint being exposed to the external environment, or when the fracture or dislocation is caused by blunt or penetrating forces sufficient to disrupt or penetrate skin, subcutaneous tissue, muscle fascia, muscle, and/ or the bone or joint. Open fractures are often contaminated by foreign material (e.g., clothing, grass, dirt, gravel), dead or devitalized tissue, and bacteria.

`Always follow the following rules in open fractures/ dislocations:

  1. Remove any gross contamination off the wound. No provisional irrigation or debridement is performed at this stage.
  2. Take photographs of the wound and dress it with wet sterile cover (the photographs are taken, so that other treating physicians do not need to uncover the wound to see it)
  3. Give patient analgesic (preferably intravenously to both control the pain and prepare for a reduction in the emergency department). Morphine is a good option.
  4. By gentle traction, reduce the fracture and correct misalignments as much as possible.
  5. Start the patient on intravenous prophylactic antibiotics.
  6. Give the patient tetanus prophylaxis if indicated.
  7. Obtain X-rays.
  8. Urgently arrange for transferring the patient to the operating room for surgical wound debridement and definite treatment of the fracture and dislocation.`

In this scenario the most appropriate initial management would be reduction of the dislocation by gentle traction after non-surgical removal of any gross contamination, and analgesia. The patient then should be started on intravenous antibiotics and receive tetanus prophylaxis.

X-ray of the joint (option E) is required pre-operatively to visualize the anatomical disruption. Wound debridement (option A) is crucial but should be performed in the operating room ideally with 1-2 hours of presentation.

How well did you know this?
1
Not at all
2
3
4
5
Perfectly
2
Q

A 72-year-old man was brought to the emergency department after he sustained a fracture of his left hip following a fall at home. Initial pain management was considered and the
operation carried out with intramedullary nail placement. Today, he is being discharged. Which one of the following is the most important management option to consider for him on
discharge?

A. Painkillers.
B. Walker.
C. Crutches.
D. Low molecular weight heparin.
E. Warfarin

A

A Painkillers.

According to guidelines provided by the Agency for Clinical Innovation (ACI), it is crucial to mobilize patients with hip fracture within 24 hours of the surgery. Pain following hip fracture should be presumed severe and continuous. Effective pain management is a primary goal for patients with a hip fracture, because immobility caused by pain has been associated with the increased risk of the following:
* Pressure ulcers
* Pneumonia
* Venous thromboembolism (VTE)

With greater pain, the likelihood of the following will increase:
* Delirium
* Sleep disturbances
* Depression

Pain control is of significant importance for early mobilization of patients who have undergone hip surgery, because the pain brought on by physical activity is the most important factor preventing from being mobile; therefore, pain control with painkillers play a crucial role in early mobilization.

Pain is often undertreated or poorly controlled in orthogeriatric patients due to:
1. Reluctance or inability of an older patient to request analgesia
2. Reluctance of medical staff to prescribe analgesia in older frail patients
3. Cognitive impairment in patients, making assessment of pain level difficult

Pain management techniques for patients with hip fracture include pharmacological approaches such as paracetamol and opioids and use of femoral nerve block.

(Options B and C) Walking aids such as crutches or walkers should be advised to provide more convenient mobilization, but they are not superior to analgesics, because with pain the patient is unlikely to become physically active and changes posture (e.g. from sitting to standing or walking).

(Option D) LMWH is also important in prevention of VTE in this patient, but this has already been started immediately after the surgery and continued during the hospital stay.

(Option E) Warfarin is not routinely used for prophylaxis of VTE in patients with hip surgery.

How well did you know this?
1
Not at all
2
3
4
5
Perfectly
3
Q

A 35-year-old woman is brought to the Emergency Department after she sustained a motor vehicle accident as a frontseat passenger, and had her left ankle injured. On examination, her vital signs are stable. The left ankle joint is laterally displaced and there is a 13-cm laceration over the joint. The left dorsal pedis pulse is barely felt and the foot is cold and pale. The condition is diagnosed as open fracture/dislocation of the ankle joint. Which one of the following is the most important step in preventing wound infection and ensuing complications?

A. Intravenous antibiotics.
B. Wound debridment.
C. Tetanus prophylaxis.
D. Reduction of the displacement.
E. X-ray of the joint.

A

B. Wound debridment.

This patient has an open fracture/dislocation of the ankle joint with vascular compromise. Open fractures/dislocations are characterized by the bone and/ or joint exposure to external environment, or when the fracture or dislocation is caused by blunt or penetrating forces sufficient to disrupt or penetrate the skin, subcutaneous tissue, muscle fascia, muscle, and/ or the bone or joint. Open fractures are often contaminated by foreign material (e.g., clothing, grass, dirt, gravel), dead or devitalized tissue and bacteria.

Always follow the following rules in open fractures / dislocations:
1. Remove any gross contamination off the wound. No provisional irrigation or debridement is performed at this stage.
2. Take photographs of the wound and dress it with wet sterile cover (the photographs are taken, so that other treating physicians do not need to uncover the wound to see it.)
3. Give patient analgesic (preferably intravenously to both control the pain and prepare for a reduction in the emergency department). Morphine is a good option.
4. By gentle traction, reduce the fracture and correct misalignments as much as possible.
5. Start the patient on intravenous prophylactic antibiotics.
6. Give the patient tetanus prophylaxis if indicated.
7. Obtain X-rays.
8. Urgently arrange for transferring the patient to the operating room for surgical wound debridement and definite treatment of the fracture and dislocation.
—–
The question, however, asks about the most important step in preventing infections and consequent complications. Debridement of all devitalized soft tissues and bone all the way down to the bone is the most crucial step in preventing the infection and the consequent adverse outcomes such as delayed healing, amputation, etc.

Option A: Prophylactic antibiotics are started prior to debridement and definite treatment, and are of great importance; however, debridement remains the cardinal step in preventing infections.

Option C: Tetanus prophylaxis should be given to the patient if indicated but this does not prevent other infections if adequate debridement is not carried performed.

Wound debridement is always crucial but should be performed in the operating room after the above measures have been undertaken; therefore, it is not a priority but an extremely
important step in preventing infections.

How well did you know this?
1
Not at all
2
3
4
5
Perfectly
4
Q

A 57-year-old construction worker man presents to your practice with the hand deformity illustrated in the accompanying photograph. He works with a drilling machine and has tingling sensation in his hand after work. He drinks 2 bottles of beers every day and even more on weekends. Which one of the following could be the most likey cause of his problem?

A. Alcohol consumption.
B. An autoimmune process.
C. Vibration injury.
D. Direct trauma.
E. Injury to the ulnar nerve.

A

A. Alcohol consumption.

The picture shows puckering of palmar skin and a nodule at the base of fourth (ring) finger which are highly suggestive of Dupuytren’s contracture (DC). DC is caused by fibrous hyperplasia of palmar fascia leading to nodular formation and contracture over the fourth and fifth fingers commonly.

This condition occurs in 10% of males over the age of 65 years particularly in those of North European descent. There is a genetic predisposition for the disease and the condition is assumed to have an autosomal dominant inheritance, but for clinical presentation to develop other factors are hypothesized to be involved.

An association between DC and some conditions have been well-established. These conditions are as follows:
-Male gender
-Alcohol Excess
-Smoking
-Liver cirrhosis
-COPD
-Diabetes mellitus
-Heavy manual labour

Of the above factors, the most common risk factors in descending order are:
-Age
-Alcohol excess
-Sex (male)
-Previous hand injuries
—–
In this man’s case, excess alcohol drinking can be the most likely contributing factor to his condition. Each bottle of beer contains 1.5 - 2 units of alcohol depending on the size and the alcohol concentration.

The pathophysiology of DC is not well-understood. Proliferation of fibrocytes and inflammatory processes are speculated to be the most likely pathophysiology. The disease is not autoimmune. No nerve is involved in DC.

How well did you know this?
1
Not at all
2
3
4
5
Perfectly
5
Q

While playing basketball, a 23-old-man landed on his left foot awkwardly and injured his ankle. He is in the emergency department now. On examination, lateral dislocation of the ankle is evident. Dorsal pedis pulse is absent and the foot looks pale and is cold to touch. Which one of the following would be the next best step in management?

A. Reduce the dislocation in the emergency department.
B. Send the patient to the operating room for reduction under general anesthesia.
C. Immobilize the joint with plaster.
D. Send the patient to the operating room for open reduction and internal fixation.
E. Angiography.

A

A. Reduce the dislocation in the emergency department.

Early reduction/alignment of a dislocation/fracture is always the most important step in management. This is even more crucial when a dislocation or fracture has caused neurovascular compromise.

In the field (prehospital), if there is evidence of neurovascular compromise such as a cold, discolored, and pulseless or insensate foot, reduction of the dislocation or alignment of the fracture should be tried after the patient is given analgesia with intravenous opiates (morphine). If morphine is not available, intravenous benzodiazepines can be used instead. Reduction and alignment is achieved by gentle in-line traction. Alignment should be maintained en route to the hospital.

In the emergency department, if the reduction/alignment has not been performed in the field, reduction should be attempted as soon as possible even before radiological examination if there is vascular compromise.

By definition, dislocation of the ankle is always considered unstable due to accompanying disruption of the lateral or medial ligaments or the tibiofibular syndesmosis. Ankle dislocation requires immediate orthopedic or podiatric consultation for surgical intervention such as internal or external fixation of any accompanying fracture and repair of capsular or ligamentous tears.

None of the other options take precedence over early reduction of the joint.

How well did you know this?
1
Not at all
2
3
4
5
Perfectly
6
Q

Which one of the following is a recognized feature of polymyalgia rheumatica?

A. Weakness of the distal muscle groups.
B. Elevated serum creatine phosphokinase (CPK).
C. An association with bronchial carcinoma.
D. Weight loss.
E. A peak incidence in the fourth decade of life.

A

D. Weight loss.

Polymyalgia rheumatica (PMR) is a relatively common inflammatory disease in elderly people (almost never seen before < 50) and a common indication for long-term steroid use. The incidence increases with advancing age. It is more common in women.

PMR has many nonspecific features and a wide range of differential diagnoses. No gold standard diagnostic test is available and the diagnosis is mainly made clinically. About 15- 20% of patients with PMR develop giant cell arteritis (GCA) and 40-50% of those with GCA are found to have associated PMR. Despite the similarities in the age of onset and some clinical manifestations, the relationship between PMR and GCA is not yet clearly established; however, these two disorders are thought to represent different manifestations of a shared disease process.

The symptoms of polymyalgia rheumatica (PMR) include pain and stiffness of the shoulder and hip girdle. The neck may be involved in some patients. The stiffness can be so severe that the patient may have great difficulty getting off the chair, turning over in bed, or raising the arms above shoulder height. Stiffness after periods of rest (gel phenomenon) as well as morning stiffness of more than 30 minutes typically occurs.

Muscle weakness is not a feature of PMR. However, this can be difficult to assess in the setting of pain, especially if symptoms are protracted and untreated, resulting in disuseatrophy. Some patients report the following systemic features:
-Low-grade fever (high, spiking fevers are rare and should prompt evaluation for underlying infection, malignancy, or vasculitis)
- Weight loss
- Malaise and/or fatigue
- Depression
- Anorexia
—–
Of the options, weight loss can be a manifestation of PMR; however, not all patients with PMR have weight loss or other systemic symptoms.

(Option A) Weakness of distal muscle groups is not a feature of PMR.

(Option B) Creatine phosphokinase elevation is associated with inflammatory processes of muscles or muscle breakdown, none of which occurs in PMR.

(Option C) There is no association between PMR and lung cancer including small cell carcinoma of the lung.

(Option E) PMR/GCA is almost never seen before the age of 50. The average age of onset is 70 years.

TOPIC REVIEW
Several diagnostic criteria for PMR exist. One set of diagnostic criteria is as follows:

-Age of onset 50 years or older
-Erythrocyte sedimentation rate ≥40 mm/h
-Pain persisting for ≥1 month and involving 2 of the following areas: neck, shoulders, and pelvic girdle
-Absence of other diseases capable of causing the same musculoskeletal symptoms
-Morning stiffness lasting ≥30 minutes
-Rapid response to prednisone (≤20 mg)

How well did you know this?
1
Not at all
2
3
4
5
Perfectly
7
Q

Michael is a 66-year-old patient of yours, who has osteoarthritis of the left knee. Six months ago and after non-pharmacological measures and paracetamol on an as-needed basis failed to relieve the pain, you started him on regular full-dose paracetamol. Today he is at your practice with the complaint that he is still suffering from severe knee pain despite
taking paracetamol. He is diabetic and has ischemic heart disease and peptic ulcer disease. Which one of the following is the most appropriate option to add to paracetamol at this stage?

A. Celecoxib.
B. Tramadol.
C. Codeine.
D. Slow-release morphine preparations.
E. Diclofenac.

A

C. Codeine.

For mild to moderate osteoarthritic pain without evidence of inflammation, pharmacological treatment starts with paracetamol. Regular full-dose of paracetamol (up to 4gr/day) even in the absence of pain is superior to taking the medication on an as-needed basis.

If the clinical response to paracetamol is not satisfactory, or if the clinical presentation of osteoarthritis is inflammatory, NSAIDs should be considered with the lowest effective dose or intermittent dosing if symptoms are intermittent. NSAIDs are associated with an increased risk of gastrointestinal problems such as dyspepsia and peptic ulcer disease and are contraindicated in patients with active peptic ulcer disease. Selective COX-2 inhibitors are preferred in these patients. The addition of prophylactic proton pump inhibitors or misoprostol is an alternative.

On the other hand, selective COX-2 inhibitors are associated with an increased risk of cerebrovascular or cardiovascular diseases and should be avoided in patients with a history of these conditions.

This patient has peptic ulcer disease and cardiovascular disease; therefore, NSAIDs and COX-2 inhibitors are inappropriate options for him.

Opiates and tramadol are drugs of choice for the treatment of patients with moderate to severe osteoarthritis, for whom NSAIDs are contraindicated or ineffective in controlling pain.

In this patient with a contraindication to both non-selective NSAIDs (e.g. ibuprofen, diclofenac, naproxen) and celecoxib, the addition of a weak opiate such as codeine will be the next best step in management.

(Option A) Celecoxib is not an appropriate option for this man because he has cardiovascular disease.

(Option B) Although tramadol is effective in the management of OA pain, either as a single therapy or added to paracetamol is associated with more adverse effects and is not considered an option at this stage. Studies showed that tramadol has more adverse effects than diclofenac.

(Option D) Strong opiates (e.g. oxycodone, morphine) are associated with more adverse effects, but about the same efficacy as weak opiates for the management of OA.

(Option E) With peptic ulcer disease, diclofenac is not an appropriate option to consider for this man.

TOPIC REVIEW

Pharmacological management of osteoarthritis of the knee
- Paracetamol – paracetamol (or other simple analgesics) are first-line pharmacological therapy for pain control. Paracetamol is safe and can be used up to 1mg every 4 hours. Patients should be advised to use paracetamol on a regular basis (even in pain-free periods) rather than as-needed because the latter often fails to adequately control the pain.

-Non-steroidal anti-inflammatory drugs (NSAIDs) – addition of NSAIDs is the next best step in management if:
1.Optimal therapy with paracetamol fails to control the pain
2.There is evidence of inflammation, such as pain that is worse with rest, nocturnal pain, and gelling of the joint (stiffness with rest).
3.COX-2 inhibitors have all the adverse effects of NSAIDs, although they seem to be less irritating to the stomach lining; therefore, better choices if there is a risk of GI bleeding

Opiate analgesics and tramadol

Opiate analgesics are used if:
-NSAIDs are contraindicated
-NSAIDs fail to control the symptoms

Opiates have a modest effect in managing moderate to severe OA pain in patients for whom paracetamol is ineffective, and who do not respond to, or have contraindications for NSAIDs. However, most of the research on opioid use has been in short-term trials and long-term efficacy has not been shown.

Intra-articular treatments
-Corticosteroids – a single injection can provide rapid relief for up to 4 weeks. The patient should be advised to rest 24 hours after the injection. This therapy can be used if the patient has an important occasion to participate in or wants to travel. Repeated injections are not as effective.

-Hyaluronan – hyaluronan is given weekly for 3 to 5 weeks. Effect sizes are small. They provide slower onset but longer-lasting (up to 12 weeks) pain relief than corticosteroids.

There is conflicting evidence of the benefit of glucosamine in the treatment of the symptoms of knee OA. There is insufficient evidence to support the benefit of preventing the progression of cartilage loss.

PRACTICE POINTS
-Regular use of full-dose paracetamol (500-1000 mg/4-6 hours, up to 4 mg /day) even when there is no pain, is superior to its use on an as-needed basis.

-Whenever NSAIDs are considered, they should be given with the minimum effective dose for 2-4 weeks and close monitoring of blood pressure, renal function, and gastrointestinal symptoms.

-NSAIDs are contraindicated in patients with active peptic ulcer disease. In those at risk of or with a history of peptic ulcer disease, either the selective COX-2 inhibitor – celecoxib should be used or non-selective NSAIDs be given with prophylactic PPIs.

-Celecoxib must be avoided in patients with a history of cardiovascular diseases such as ischemic heart disease, cerebrovascular disease, etc

How well did you know this?
1
Not at all
2
3
4
5
Perfectly
8
Q

A 67-year-old woman presents to your clinic with complaints of painful hips and shoulders and headache. She relates that muscle pain is felt in both arms and hips. She feels her shoulders are stiff and her hips cannot move when she wakes up. It almost takes 40 minutes of painful physical activity for the stiffness to relent. Her appetite is decreased and she
has lost 5 kg. “To make me even more miserable, the terrible headaches have just been started”, she mentions. The headache is felt on the right side and is throbbing with no response to analgesics. On physical examination, she is noted to have bilateral painful restricted movements of shoulder and hip joints. Her right temple is tender to touch. She is
not febrile and the rest of the exam is inconclusive. Which one of the following would be the most appropriate next step in management?

A. Check serum creatine kinase (CK) level.
B. Check the rheumatoid factor.
C. Check ESR.
D. Perform a muscle biopsy.
E. Star the patient on prednisolone.

A

C. Check ESR.

The clinical picture is highly suggestive of two closely-associated clinical syndromes, polymyalgia rheumatica and giant cell (temporal) arteritis (GCA). The etiology of these two syndromes is unknown. Either of these syndromes can develop solitarily, although in 15-20% of cases both are present. If so, polymyalgia rheumatica invariably precedes temporal arteritis.

The patients are usually older (the 60s or 70s). These conditions are very rare among those younger than 50 years. Women are more commonly affected.

Polymyalgia rheumatica presents with pain and stiffness of the shoulders and hips symmetrically. The cervical spine may be involved. There is marked stiffness usually early in the morning lasting over 30 minutes. The symptoms tend to subside or at least relieve by the end of the day. Constitutional symptoms such as malaise, low-grade fever, anorexia, and
weight loss may be present.

Giant cell arteritis presents with headaches often unresponsive or poorly responsive to analgesics, scalp tenderness, jaw claudication, and weak or absent pulsation of the temporal artery. Not all patients with CGA present with headaches.

When these two conditions are suspected, the next best step in management would be measuring the erythrocyte sedimentation rate (ESR). With an elevated ESR (>40), it is prudent that corticosteroids be started immediately to prevent permanent visual loss. With visual problems at presentation, intravenous corticosteroids (methylprednisolone) replace the oral route for the first 3 days of therapy.

(Options A and D) Creatine kinase levels or muscle biopsy would come first if myositis was suspected.

(Option B) Serologic tests, such as rheumatoid factor (RF), antinuclear antibodies (ANA), and cyclic citrullinated peptide (CCP) antibodies are typically negative.

(Option E) Corticosteroids (e.g. prednisolone) are indicated if ESR is >40mm/h.

How well did you know this?
1
Not at all
2
3
4
5
Perfectly
9
Q

Which one of the following cannot be associated with HLA- B27?

A. Acute anterior uveitis.
B. Ankylosing spondylitis.
C. Reactive arthritis.
D. Behçet disease.
E. Psoriatic arthritis.

A

D. Behçet disease.

HLA-B27 is found in almost 90% of patients with seronegative spondyloarthropathies.

Seronegative spondyloarthropathies include:
-Ankylosing spondylitis (AS)
-Reactive arthritis
-Psoriatic arthritis
-Inflammatory bowel disease

Anterior uveitis is another possible finding in seronegative spondyloarthropathies; hence, from another standpoint, it could be said that those with anterior uveitis may suffer from a seronegative spondyloarthropathy and consequently be HLA-B27 positive.

Behçet disease is a diffuse vasculitis of unknown origin. The disease is characterized by a triple-symptom complex of:
1. Recurrent oral aphthous ulcers
2. Recurrent genital ulcers
3. Uveitis

Behçet disease has no association with HLA-B27

How well did you know this?
1
Not at all
2
3
4
5
Perfectly
10
Q

A 67-year-old man presents to your office with complaints of reduced dexterity of his left hand and difficulty grasping objects. His hand is shown in the following photograph. Which one of the following investigations will confirm the diagnosis?

A. X-ray of the hand.
B. MRI.
C. Ultrasonography of the hand.
D. CT scan of the hand.
E. Blood sugar level.

A

C. Ultrasonography of the hand.

The picture shows palmar skin puckering and a nodule at the base of the fourth finger, indicative of Dupuytren contracture. The diagnosis is almost always made clinically and no imaging is needed. However, ultrasonography can demonstrate thickening of the palmar fascia and the presence of a nodule. At cases, ultrasonography of the hand is considered if the patient is planned for injection of corticosteroids directly into the cord and avoiding injecting into the flexor tendon.

Since there is an association between Dupuytren contracture and diabetes mellitus, blood sugar level can be measured if the physician is suspicious of diabetes based on history and/or clinical findings. X-ray, CT scan or MRI is almost never indicated.

Treatment options for Dupuytren contracture:
-Physiotherapy (for mild cases): e.g. heat and ultrasonographic waves, physical therapy, custom splint or brace for finger stretching
-Occupational therapy
Intralesional corticosteroid injection
-Collagenase injection
-Other measures: 5-fluorouracil, imiquimod, botulinum toxin, and hyperbaric oxygen
-Surgery: surgery is indicated if MCP contracture is more than 30° or there is contracture of PIP

How well did you know this?
1
Not at all
2
3
4
5
Perfectly
11
Q

A 32-year-old man presents to your office with complaint of low back pain in the setting of ankylosing spondylitis (AS), which he has had for the past 5 years. He has been on treatment with naproxen 500 mg, 3 times a day, for the past few years with no significant improvement. An X-ray of the lumbosacral area is obtained and is shown in the following
photograph. Which one of the following is the most appropriate management option for this patient?

A. Start him on sulfasalazine.
B. Start him on methotrexate.
C. Start him on infliximab.
D. Start him on cyclosporine.
E. Double the dose of naproxen.

A

C. Start him on infliximab.

Non-steroidal anti-inflammatory drugs (NSAIDs) are first-line therapy for symptomatic patients with AS. Recent studies suggest that regular NSAID use in AS slows radiographic progression more than on-demand use. An individualized assessment of the risk of long-term NSAID use should be made in consultation with the rheumatologist before long-term daily NSAID use is recommended.

Tumour Necrotizing Factor (TNF) inhibitors such as infliximab are used when there is not adequate response to at least two different NSAIDs. TNF inhibitor therapy has strikingly improved the quality of life in over two-thirds of the patients. These agents are listed on the Pharmaceutical Benefits Scheme (PBS) for active AS not adequately responsive to exercise and NSAIDs, but not for non-radiographic axial AS, meaning that these drugs are not indicated where the X-ray changes of sacroiliitis are absent. Randomized controlled trials, however, suggest better clinical responses in early non-radiographic disease.

This patient has not adequately responded to maximum dose of naproxen (an NSAID). On the other hand, his lumbosacral X-ray shows the classic bamboo spine (squaring and fusion of lumbosacral vertebrae) seen in AS; therefore, he will be an appropriate candidate to be started on infliximab as the most appropriate next step in management.

(Options A and B) Sulfasalazine and methotrexate are not indicated where there is axial involvement.

(Option D) Cyclosporine has not shown effective in management of axial AS.

(Option E) Doubling the dose of naproxen will exceed the maximum dose of this drug and is associated with significant adverse effects and probably no improved outcome.

NOTE - If there was an option suggesting switching to another NSAID, it would be the correct answer because some patients unresponsive to one NSAID may improve on another.

How well did you know this?
1
Not at all
2
3
4
5
Perfectly
12
Q

A 68-year-old woman presents to your practice complaining of pain in her right hip. The pain started 2 days ago and has escalated since. She refuses to bear weight on the affected side, as this brings on severe pain. The patient denies any history of osteoarthritis or other joint diseases, but admits to long-term use of oral corticosteroids for Behçet disease. On examination, all movements of the right hip joint are diminished and very painful. Which one of the following would be the most likely diagnosis?

A. Gout.
B. Avascular necrosis of the femoral head.
C. Osteoarthritis of the hip joint.
D. Septic arthritis.
E. Bony metastasis from a primary cancer.

A

B. Avascular necrosis of the femoral head.

There are many conditions that can cause hip or groin pain in adults. The following 2 are the most common causes in adults:
- A flare of a chronic inflammatory condition such as rheumatoid arthritis
- Degenerative changes of the hip joint, such as in osteoarthritis

Other causes, especially in the elderly may include:
- Fracture of the femoral head or neck
- Degenerative spondylosis of lumbosacral spine (neurogenic claudication)
- Arterial occlusions (vascular claudication)
- Polymyalgia rheumatica
- Trochanteric bursitis
- Bony metastases from a primary cancer
- Atraumatic avascular necrosis of the femoral head

Atraumatic avascular necrosis of the femoral neck, although not a very common condition, should always be suspected in an elderly patient, particularly if the following risk factors are present in the history:
-Long-term steroid use
-Chronic alcohol use and resulting liver disease
-Sickle cell disease
-Past hip fracture
—–
Of the given options, considering the history of long-term steroid use, avascular necrosis of the femoral head seems more likely.

The initial step in diagnosis would be an X-ray of the hip. Typical finding is variable density of the femoral head.

Sonography can be used to exclude joint effusion due to an acute inflammation such as in a flare up of rheumatoid arthritis or septic arthritis. The most accurate diagnostic tool would be an MRI with characteristic ‘geographic’ subchondral lesions. It is often considered if the diagnosis remains equivocal.

(Option A) Gout can cause acute inflammation and pain of the joint, but the hip joint is unlikely to be affected by this disease.

(Option C) Sudden onset of the hip pain, without any previous history, is a very unlikely to be caused by osteoarthritis (OA) of the hip. The history of OA often includes insidious onset and progression to more severe states over a long period of time.

(Option D) Septic arthritis is a serious condition that can lead to complete joint destruction and permanent disability within hours if untreated, and needs to be excluded; however, it is not the most common diagnosis in this case scenario. The patient with septic arthritis is expected to be more systematically ill, but the range of motions of the affected hip joint
would be painfully limited, just such as this patient.

(Option E) Bony metastases from a primary cancer does not seem to be a likely diagnosis, as a cancer so advanced to metastasize to distant sites, is expected to have presented with sign and symptoms of the primary tumor earlier. The examination is mentioned to be unremarkable otherwise.

How well did you know this?
1
Not at all
2
3
4
5
Perfectly
13
Q

A 35-year-old man presents to your GP clinic with severe back pain for several years. The pain is worse on waking up in the morning. Pain and stiffness make him stay in bed for at least 30 minutes. The pain ameliorates by walking and physical activity throughout the day. His condition has deteriorated recently despite being on the maximum dose of naproxen. An X-ray of his lumbosacral spine is shown in the accompanying photograph. Which one of the following would be the most appropriate next step in management?

A. Sulfasalazine.
B. Methotrexate.
C. Adalimumab.
D. Aspirin.
E. Prednisolone.

A

C. Adalimumab.

The X-ray shows severe deformities of the lumbosacral spine evident by the fusion of vertebrae resulting in the characteristic bamboo spine, a hallmark radiographic feature in ankylosing spondylitis (AS).

Non-steroidal anti-inflammatory drugs (NSAIDs) are first-line therapy for symptomatic patients with AS; however, when the disease is rapidly progressive and/or there is not adequate response despite the maximum dose of NSAIDs, biological disease-modifying anti-rheumatic drugs (bDMARDs) are next step in management.

Of these drugs, Tumor Necrotizing Factor (TNF) inhibitors have strikingly improved the quality of life in more than two-thirds of the patients. These agents are indicated for active AS not adequately responsive to exercise and NSAIDs with radiographic axial AS (as in this case), but not for non-radiographic axial AS, meaning that these drugs are not indicated where the X-ray changes typical for AS are absent. Randomized controlled trials, however, suggest better clinical responses in early non-radiographic disease. Infliximab, etanercept, adalimumab, and golimumab are TNF inhibitors approved for the treatment of AS when indicated.

With inadequate response to NSAIDs and characteristic X-ray findings, this patient is eligible for treatment with adalimumab as the next best step in management.

Traditional DMARDs such as methotrexate, azathioprine, and sulfasalazine have little effect on spinal disease, but can be useful for associated peripheral arthritis. Other biologic agents available for rheumatoid arthritis in Australia have not been effective for the treatment of AS.

(Options A and B) Sulfasalazine and methotrexate are not effective treatments when there is axial involvement.

(Option D) If among the options, a trial of a second NSAID would be the correct answer because one of the criteria required for bDMARD to be covered by the pharmaceutical benefits scheme (PBS) is the failure of at least two NSAIDs before the commencement of bDMARDs. Aspirin is a salicylate, not an NSAID.

(Option E) Systemic steroids may be used for extra-spinal presentations of AS such as uveitis, inflammatory bowel disease (IBD), etc. but are not indicated nor effective for the treatment of spinal involvement

How well did you know this?
1
Not at all
2
3
4
5
Perfectly
14
Q

A 51-year-old man presents to your practice for back pain. He was diagnosed with ankylosing spondylitis 20 years ago and has had mild back pain all this time. He had received no
medical treatment and has been on an exercise program on an on-and-off basis. The patient suffers from peptic ulcer disease and has been treated for the condition several times.
Which one of the following is the most appropriate next step in management?

A. Paracetamol.
B. Naproxen.
C. Methotrexate.
D. Sulfasalazine.
E. Infliximab.

A

Correct Answer Is A

First-line management of ankylosing spondylitis (AS) is NSAIDs. Regular use of NSAIDs has been shown to reduce the symptoms as well as the rate at which skeletal changes
occur. This patient has been diagnosed with AS 20 years ago and has not received any medication. On the other hand, mild low back pain is the patient’s current complaint.
Without gastric problems, NSAIDs (naproxen) was the drug of choice to consider first, but with peptic ulcer disease and because the pain is mild, simple analgesics such as paracetamol should be tried first in an attempt to alleviate the patient’s pain without significantly increasing the risk of his peptic ulcer disease deteriorate.

NOTE - In general, simple analgesics and opioids are considered where there is residual pain despite NSAID use or when NSAIDs are contraindicated.

(Option B) Despite the increased risk, NSAIDs may be considered at the lowest doses possible, once simple analgesics proved ineffective in pain control. COX2-selective NSAIDs
such as celecoxib are preferred over non-selective NSAIDs if the patient does not have cardiovascular or cerebrovascular diseases.

(Options C and D) Methotrexate and sulfasalazine may occasionally be considered for the treatment of extra-spinal manifestation of AS.

(Option E) Infliximab is a tumor necrotizing factor (TNF) inhibitor approved to use in patients with axial involvement with inadequate response to a maximum dose of NSAIDs or in
those with severely progressive disease.

How well did you know this?
1
Not at all
2
3
4
5
Perfectly
15
Q

An 80-year-old woman sustains a fall in the bathroom and is brought to the emergency department by her daughter. She has severe pain and decreased range of motion in all
directions in the right hip. The right leg appears shorter than the left and is externally rotated. A bedside portable X-ray of the pelvis is taken which is shown in the following
photograph. Which one of the following is the most appropriate management option for this patient?

A. Intramedullay nail placement.
B. External fixation.
C. Hip arthroplasty.
D. Analgesics and bed rest.
E. Traction.

A

Correct Answer Is C

The X-ray shows an intracapsular (Garden) fracture of the right femoral neck. The fracture can be subcapital, transcervical, or basicervical. Only such fractures can be considered genuine femoral neck fractures.

Garden classification of intracapsular (subcapital) fractures of the femoral neck is as follows:

Garden type I: incomplete fracture with valgus impaction
Garden type II: Complete fracture without displacement
Garden type III: Complete fracture with partial displacement of the fracture fragments
Garden type IV: Complete fracture with total displacement allowing the femoral head to rotate back to an anatomic position
In reality the distinction between classes is difficult; however, complete fracture and displacement of the femoral head makes Garden type III (or IV) femoral neck fracture the most
likely diagnosis.
Garden types I and II femoral neck fractures are surgically stabilized with closed reduction and internal fixation. Garden types III and IV are controversial in the type of implant used
for treatment. In younger patients, closed or open reduction is recommended. In less active older patients, however, prosthetic replacement is recommended.
None of other options are superior to hip arthroplasty.

How well did you know this?
1
Not at all
2
3
4
5
Perfectly
16
Q

Examination of an aspirate from an acutely painful swollen knee is as follows:
White blood cell count: 4100 (<2000μL)
Red blood cells: few
Crystals: negative
Culture: no growth
Which of the following is the most accurate interpretation of the results?

A. Traumatic tap.
B. Inflammation.
C. TB arthritis.
D. Viral arthritis.
E. Gonococcal arthritis.

A

Correct Answer Is B

Biochemically, the synovial fluid is an ultrafiltrate of plasma across the synovial membrane enriched with various compounds produced by the synoviocytes. In normal physiological conditions, the biochemical composition of
the synovial fluid is similar to that of plasma. In pathological conditions, laboratory evaluation of synovial fluid provides information about the pathology of the disease affecting the joint (e.g., arthritis).

Similar to other body fluids, common laboratory evaluation of synovial fluid includes the following three steps:
Physical examination of synovial fluid (e.g., appearance, including color, viscosity, and other physical characteristics)
Chemical analysis (e.g., glucose, total protein level, uric acid)
Microscopic evaluation (e.g., cell count and differential count, crystal identification)

Based on physical and laboratory findings, synovial fluid can be categorized as:
Normal
Non-inflammatory
Inflammatory
Septic
Hemorrhagic
The following table summarizes the characteristic feature of synovial fluid in different conditions: (See photo)

Although a cell count of 4800 in the aspirate analysis could be seen in both inflammatory and infectious conditions, a very high cell count of >50,000 or even >100,000 is more specific for infectious causes (read the Exam Tip below). Furthermore, a negative culture does not exclude infection but makes it less likely.
In practice, a cell count of 2000-50,000 is the typical finding in inflammatory joint diseases such as rheumatoid arthritis, lupus arthritis, gout, and pseudogout. The absence of crystals, however, makes crystal-induced
inflammation unlikely.

(Option A) Although a traumatic tap results in blood in the synovial fluid, other features such as elevated white cell count are absent unless a concomitant condition is present. A few red blood cells (RBCs) are usually present
even in a normal synovial fluid.

(Option C) In TB arthritis, synovial fluid analysis is usually not helpful. The white cell count can be high or low with the dominance of either neutrophils or lymphocytes and no other specific diagnostic features. In fact, this
aspirate analysis can also be caused by TB as well, yet an inflammatory condition is more likely.

(Option D) Viral arthritis presents with a high white cell count with a predominance of polymorphonuclears (PMNs), but the cultures are negative.

(Option E) A white cell count of 4100, makes gonococcal arthritis less likely. Gonococcal arthritis often presents with a white cell count of 34000-68000

EXAM TIP

Remember the following tips for the exam:
The WBC count of a normal joint aspirate is < 200.
With degenerative diseases (e.g., OA) or trauma, the WBC count of joint fluid is 200-2000.
Typical inflammatory joint fluid either with the crystalline or non-crystalline disease has a WBC of 2000-50,000 (more commonly 5,000-50,000)
The typical WBC count of joint fluid in septic arthritis is usually >50,000; however, septic arthritis should be considered a possibility in every joint aspirate with WBC>5000, especially if the arthritis is monoarticular and
the patient has already received antibiotics.
It is not common for gout and pseudogout to present with WBC>50,000. If so, co-infection is a possibility to consider.

How well did you know this?
1
Not at all
2
3
4
5
Perfectly
17
Q

A 37-year-old man is brought to the Emergency Department after he sustained a motor vehicle accident. Upon arrival, a quick review excludes any life-threatening internal injuries.
He has a deformed right leg highly indicative of a fracture. There is a bleeding laceration over the deformity. An X-ray shows fractures of the tibial and fibula. As the initial
management, intravenous fluid and antibiotics are started. Which one of the following is the most appropriate next step in management?

A. External fixation.
B. Internal fixation.
C. Tetanus prophylaxis.
D. Wound debridment.
E. Wound closure.

A

Correct Answer Is C

The vignette describes a typical case of open fracture of the right leg. Open fractures are characterized by the fracture exposure to the environment trough a breach of the skin and
underlying soft tissue. Such fractures are particularly at increased risk of infection that can compromise the fracture and healing process.

Always follow the following rules in open fractures / dislocations
**Remove any gross contamination off the wound. No provisional irrigation or debridement is performed at this stage.
**Take photographs of the wound and dress it with wet sterile cover (the photographs are taken, so that other treating physicians do not need to uncover the wound to see
it.)
**Give patient analgesic (preferably intravenously to both control the pain and prepare for a reduction in the emergency department). Morphine is a good option.
**By gentle traction, reduce the fracture and correct misalignments as much as possible.
**Start the patient on intravenous prophylactic antibiotics.
**Give the patient tetanus prophylaxis if indicated.
**Obtain X-rays.
**Urgently arrange for transferring the patient to the operating room for surgical wound debridement and definite treatment of the fracture and dislocation.

There is no comment regarding significant misalignment. The patient’s wound should be covered by wet dressing after photographs are taken (if possible). He also has been started
on intravenous antibiotics; therefore, the next best step for him would tetanus prophylaxis if indicated.

(Options A and B) Fixation of fractures is performed in the operating room after surgical irrigation and wound debridement.

(Option D) Wound debridement including surgical removal of all devitalized soft tissues and bone is a crucial step in management of any open fracture/dislocation because
infections may ensue and result in poor outcomes. However, debridement should be performed in the sterile environment of the operating room, not in the non-sterile environment
of the Emergency Department where more exploration of the wound can lead to increased risk of infection.

(Option E) Wound closure can be performed (if not contraindicated such as in highly contaminated wounds or delayed presentation) after vigorous irrigation and debridement of the
wound and fixation of the fracture.

How well did you know this?
1
Not at all
2
3
4
5
Perfectly
18
Q

You are visiting a 75-year-old woman in your practice, who is complaining of bilateral pain and stiffness in her shoulders and hips. She explains that the pain and stiffness are worst
on waking, but becomes better after 40 to 60 minutes of physical activity. On examination, there is no remarkable finding except generalized stiffness of her hip and shoulder
girdles. Which of the following investigations is most consistent with the diagnosis?

A. Elevated serum calcium.
B. Elevated erythrocyte sedimentation rate (ESR).
C. Positive rheumatoid factor (RF).
D. Increased serum alkaline phosphatase level.
E. Joint space narrowing and decreased bone density on X-rays of the shoulders and hips.

A

Correct Answer Is B

Pain and morning stiffness of hip and shoulder girdles taking 40-60 minutes of activity to relieve is consistent with polymyalgia rheumatic as the most likely diagnosis.

Polymyalgia rheumatica is an inflammatory rheumatic condition characterized clinically by aching and morning stiffness in the shoulders, hip girdle, and neck. The condition can be
associated with giant cell (temporal) arteritis (GCA). The 2 disorders are thought to represent different manifestations of a shared disease process.

Polymyalgia rheumatica is almost exclusively a disease of adults over the age of 50, and 2- to 3-times more common in women than men.
The prevalence increases with age and
the average age at diagnosis is over 70 years.

Clinical manifestations of polymyalgia rheumatica include:
**The subacute or chronic onset of aching and morning stiffness in the shoulders, hip girdles, neck, and torso (usually symmetric)
**Morning stiffness that typically lasts more than 30 minutes
**Pain – shoulder pain is more common at presentation (70-95%) than hip and neck (50-70%). The pain is worse with movement
**Decreased range of motion
**Synovitis, tenosynovitis, and bursitis - ~50% of patients can exhibit distal musculoskeletal involvement; some patients may have swelling and pitting edema of the hands,
wrists, ankles, and top of the feet. Tenosynovitis can also cause carpal tunnel syndrome, which occurs in approximately 10 to 15% of patients with polymyalgia rheumatica
**Subjective weakness - Muscle strength is usually normal, but weakness is frequently reported by patients and may be a problem diagnostically due to diminished effort on
examination because of pain, or clinically because of disuse atrophy. Thus, careful testing is required if weakness is reported.
**Systemic manifestations - Up to 40% of patients may have malaise, fatigue, depression, anorexia, weight loss, and fever

NOTE – Muscle tenderness is not a prominent feature despite what the name implies. If present, tenderness is more likely to be due to synovial or bursal inflammation than
muscle involvement.

The most characteristic finding in polymyalgia rheumatica (and giant cell arteritis) is an elevated erythrocyte sedimentation rate (ESR) that is above 40mm/h. According to some
studies, an elevated ESR can be seen in 78-93%. ESR may exceed 100mm/h in some patients.

(Option A) Serum calcium is unaffected in polymyalgia rheumatica.

(Option C) Serologic tests, such as rheumatoid factor (RF), antinuclear antibodies (ANA), and cyclic citrullinated peptide (CCP) antibodies are typically negative.

(Option D) Some patients may have some rise in their liver enzymes, especially alkaline phosphatase; however, this is more common in patients with giant cell arteritis than
polymyalgia rheumatica alone.

(Option E) Joints are not affected in polymyalgia rheumatica. No X-ray abnormalities are expected

References
* UpToDate- Clinical Manifestations and diagnosis of polymyal

How well did you know this?
1
Not at all
2
3
4
5
Perfectly
19
Q

A 74-year-old woman presents to the emergency department with sudden-onset left visual loss that occurred within a few hours. She also mentions having left-sided headaches for
the past few days and hip and shoulder pain and stiffness for 7 months. She has tried over-the-counter painkillers for her headache with no response. Which one of the following is
the most appropriate next step in management?

A. Muscle biopsy.
B. CT scan of the head.
C. MRI of the head.
D. Erythrocyte sedimentation rate (ESR).
E. Ocular massage.

A

Correct Answer Is D

Sudden-onset visual loss and headache in a patient over the age of 50 is very likely to be giant cell (temporal) arteritis until proven otherwise. Additionally, this patient has had hip
and shoulder pain, suggestive of polymyalgia rheumatica as a strong association.

Giant cell arteritis (GCA) and polymyalgia rheumatica are closely associated syndromes with unknown etiology. GCA and polymyalgia rheumatica can develop alone, although in
20% of cases both are present (as in this case)
Polymyalgia rheumatica presents with pain and stiffness of shoulders and/or hips often symmetrically. Neck pain and/or stiffness may be present. Shoulder pain is the presenting
symptom in 70-95% of patients. Hip and/or shoulder pain is present in 50-70% of patients.

Tenosynovitis and bursitis may be present in 50% of patients. A decreased range of
motion can be found on examination. A subjective sense of muscle weakness may be reported by the patient despite the fact that muscles are not involved. Constitutional
symptoms such as fever, weight loss, malaise, fatigue, and anorexia are present in 40% of patients.

GCA presents with headaches often unresponsive or poorly responsive to analgesics, scalp tenderness, jaw claudication, and weak or absent pulsation of the temporal artery. A new headache occurs in at least 2/3 of patients. The headache may have a variety of presentations in GCA, but being new-onset is the critical feature. In its classic form, the headache of GCA is located in temporal regions, but it may also occur in frontal or occipital areas or may be generalized. Headache is the chief complaint in most patients, but in some, it may be absent or elicited by direct questioning.
Systemic symptoms of GCA include fever, fatigue, and weight loss. Up to 50% of patients have a fever that is often low-grade but can exceed 39°C in 15% of patients which can be
mistaken for infection. Importantly, in approximately 10% of patients, constitutional symptoms are the only manifestations of GCA.

NOTE – in patients with polymyalgia rheumatica, ongoing monitoring for symptoms or physical findings suggestive of GCA is recommended. Evaluation including biopsy should be
performed if symptoms of GCA develop, even if patients are on corticosteroids.

Once GCA/polymyalgia rheumatica is suspected on clinical grounds, ESR is the most appropriate next step in management. An elevated ESR above 40mm/h is the most
characteristic finding in GCA/polymyalgia rheumatica. With an elevated ESR, immediate treatment with systemic corticosteroids should be started and a temporal artery biopsy for a
definite diagnosis arranged.

(Option A) Although some patients may complain of muscle pain and tenderness, muscles are not involved in GCA/polymyalgia rheumatica. If present, tenderness is more likely to
be due to synovial or bursal inflammation than muscle involvement. Muscle biopsy has no role in the diagnosis of these conditions.

(Options B and C) Imaging studies such as CT scans or MRIs are not routinely used for diagnosis of GCA/polymyalgia rheumatica unless the diagnosis is equivocal and other
diagnoses are considered.

(Option E) ocular massage is used initially for urgent management of retinal artery occlusion that not the case here

How well did you know this?
1
Not at all
2
3
4
5
Perfectly
20
Q

A 76-year-old woman presents with painful and stiff shoulders that she has had for the past 2 months. The stiffness is worse in the morning and improves after 30-60 minutes of
physical activity. She denies headaches, visual problems, or symptoms such as fever or weight loss. Initial laboratory tests results are:

RBC: 5x10^6 cells/mm3 (4.5-6.5x10^6)
Hb:115g/L (130-180)
MCV: 85 fL (76-96)
WCC: 9000 cells/mm3 (N: 4000-11000)
Platelet: 500,000/mm3 (N: 150,000-400,000)

Metabolic panel: normal
ESR: 65mm/h
CRP: Positive
RF: negative
ANA: negative

Which of the following would you consider for treating this patient?

A. Paracetamol.
B. Prednisolone.
C. Antibiotics.
D. Non-steroidal anti-inflammatory drugs (NSAIDs).
E. Physical therapy.

A

Correct Answer Is B

The painful stiff shoulder and/or hip girdle in a person over the age of 50 years is mostly consistent with polymyalgia rheumatica as the most likely diagnosis.

Polymyalgia rheumatica is an inflammatory rheumatic condition characterized clinically by aching and morning stiffness in the shoulders, hip girdle, and neck. The condition can be
associated with giant cell (temporal) arteritis (GCA). The two disorders are thought to represent different manifestations of a shared disease process.

Polymyalgia rheumatica is almost exclusively a disease of adults over the age of 50, and 2- to 3-times more common in women than men. The prevalence increases with age and
the average age at diagnosis is over 70 years.
Polymyalgia rheumatica presents with pain and stiffness of shoulders and/or hips often symmetrically. Neck pain and/or stiffness may be present. Shoulder pain is the presenting
symptom in 70-95% of patients. Hip and/or neck pain is present in 50-70% of patients. Tenosynovitis and bursitis may be present in 50% of patients. A decreased range of motion
can be found on examination. A subjective sense of muscle weakness may be reported by the patient despite the fact that muscles are not involved. Constitutional symptoms such
as fever, weight loss, malaise, fatigue, and anorexia are present in 40% of patients.

The most characteristic finding in polymyalgia rheumatica (and giant cell arteritis) is an elevated erythrocyte sedimentation rate (ESR) that is above 40mm/h. According to some
studies, an elevated ESR (and CRP) can be seen in 78-93% .ESR may exceed 100mm/h in some patients.

Normocytic normochromic anemia is seen in some patients. WCC is normal, but platelet count can be raised, as platelets are acute phase reactants and can be elevated in
inflammatory conditions. Serologic tests, such as rheumatoid factor (RF), antinuclear antibodies (ANA), and cyclic citrullinated peptide (CCP) antibodies are typically negative.
Some may have an increase in their liver enzymes, especially alkaline phosphatase; however, this is more common in patients with giant cell arteritis than polymyalgia rheumatica
alone.

Corticosteroids are recommended as the initial therapy in patients with polymyalgia rheumatica. The primary objective of treatment is improvement in symptoms. Therapy is mostly
symptomatic and has not been proven to improve prognosis or prevent progression to giant cell arteritis. Most patients experience dramatic improvement in symptoms within days
of therapy.

Prednisone/prednisolone 0-20mg, daily in the morning, is the recommended regimen. Therapy is often required for 2 to 3 years. The minimum duration of therapy is unlikely to be
less than 12 months. Long-term use of corticosteroid use in such patients is associated with corticosteroid-induced bone loss and osteoporosis. Prophylactic calcium/vitamin D
supplementation is recommended and bisphosphonates (e.g. alendronate) may be indicated for prevention.

In patients with CGA, daily doses of 40-60mg are required. The addition of aspirin is recommended. For patients with visual symptoms, who present early, a 3-day course of
intravenous prednisolone, followed by oral therapy is the management of choice.

NOTE – Response to treatment is monitored with ESR and CRP.
(Option A) Polymyalgia rheumatica is an inflammatory condition. Paracetamol is unlikely to be of considerable benefit in patients with polymyalgia rheumatica.

(Option C) Antibiotics have no role in the management of polymyalgia rheumatica and/or GCA.

(Option D) NSAIDs have been tried as corticosteroid-sparing alternatives, but can be associated with significant adverse effects. NSAIDs are occasionally used in patients on very
low doses of corticosteroids, who also require analgesic or anti-inflammatory therapy for other disorders, such as osteoarthritis or rotator cuff tendonitis.

(Option E) Physical therapy is recommended as adjunctive therapy in patients who have difficulty regaining good mobility and a full range of motion despite control of the
inflammatory process.

How well did you know this?
1
Not at all
2
3
4
5
Perfectly
21
Q

A 73-year-old woman presents to your practice with a complaint of shoulder pain for the past 8 months. The pain is mostly felt in both shoulders and is aching and vague in nature.
She also has shoulder girdle stiffness in the morning, often taking 30-60 minutes of physical activity and sometimes a hot bath to improve. She denies any fever, weight loss,
headache, or visual symptoms. Past medical history is significant for osteoporosis, for which she is on alendronate and supplemental calcium and vitamin D. Laboratory tests show
normocytic, normochromic anemia and an erythrocyte sedimentation rate (ESR) of 100mm/h. Which one of the following is the most appropriate next step in management?

A. Corticosteroids.
B. Non-steroidal anti-inflammatory drugs (NSAIDs).
C. Paracetamol.
D. Methotrexate.
E. Infliximab.

A

Correct Answer Is A

The clinical picture is highly suggestive of polymyalgia rheumatica an inflammatory disease of uncertain etiology that can be associated with giant cell arteritis in some patients. If both conditions are present polymyalgia rheumatica invariably precedes GCA.
As in this patient, polymyalgia rheumatica presents with pain and stiffness of the shoulders and hips in a symmetrical fashion. The cervical spine may be involved. There is marked stiffness usually early in the morning. The symptoms tend to subside or at least relieve by the end of the day. Constitutional symptoms such as malaise, low-grade fever, anorexia, and weight loss may be present in 40% of patients.

Corticosteroids are recommended as the initial therapy in patients with polymyalgia rheumatica. The primary objective of treatment is the improvement in symptoms.
Prednisone/prednisolone 10-20mg, daily in the morning is the recommended regimen. Therapy is often required for 2 to 3 years. The minimum duration of therapy is unlikely to be
less than 12 months. Long-term use of corticosteroid use in such patients is associated with corticosteroid-induced bone loss and osteoporosis. In this patient with pre-existing
osteoporosis, long-term corticosteroid therapy may result in the deterioration of bone loss; nonetheless, there is no alternative effective therapy for myalgia rheumatica to replace
corticosteroids.
This patient should also be started on preventive measures to decrease the adverse effects of corticosteroids on bone, if the duration of therapy is predicted to exceed one month:
**Weight-bearing exercises where possible
**Adequate calcium intake
**Measurement and addition of vitamin D if indicated
**Monitoring bone density from the outset of therapy
**Bisphosphonates if indicated

(Option B) NSAIDs have been tried as corticosteroid-sparing alternatives, but can be associated with significant adverse effects. NSAIDs are occasionally used in patients on very low doses of corticosteroids who also require analgesic or anti-inflammatory therapy for other disorders, such as osteoarthritis or rotator cuff tendonitis. NSAIDs alone are unlikely to be useful for the treatment of polymyalgia rheumatica.

(Option C) Polymyalgia rheumatica is an inflammatory condition. Paracetamol is unlikely to be of considerable benefit in patients with polymyalgia rheumatica.

(Option D) Methotrexate alone is not shown to be effective in managing polymyalgia rheumatica Addition of methotrexate to corticosteroids may be used to reduce the dose of corticosteroids in patients who are at increased risk of corticosteroid-induced adverse effects such as osteopenia/osteoporosis. This has been suggested by some but not all
studies. In general, methotrexate is rarely used in patients with polymyalgia rheumatica without GCA, because low doses of corticosteroids are often sufficient to control the
disease.

(Option E) The use of TNF inhibitors, such as infliximab and etanercept, is effective in the treatment of polymyalgia rheumatica as an alternative to corticosteroids

How well did you know this?
1
Not at all
2
3
4
5
Perfectly
21
Q

A 70-year-old man presents with complaints of headache and blurry vision in his right eye. He also mentions a history of shoulder pain that has been bilateral and associated with
morning stiffness of at least 30 minutes for the past few months. Which one of the following investigations is most important to establish the diagnosis?

A. Erythrocyte sedimentation rate (ESR).
B. ANA.
C. Temporal artery biopsy.
D. CT scan of the head.
E. Evoked visual potential.

A

Correct Answer Is C

The clinical picture of headache and visual impairment in a patient over the age of 50 is highly suggestive of giant cell arteritis. The preceding history of shoulder pain and stiffness
indicates polymyalgia rheumatica and makes the diagnosis more certain.

When CGA is suspected the initial step is measuring the erythrocyte sedimentation rate (ESR). An elevated ESR is the most characteristic finding in patients with GCA/polymyalgia
rheumatica. An ESR level of less than 40mm/h makes CGA/polymyalgia rheumatica an unlikely diagnosis while measurings above 40mm/h prompt treatment with corticosteroids to
prevent permanent visual loss. Arrangements then should be made to perform a temporal artery biopsy. Superficial temporal artery biopsy is the criterion standard for diagnosis
of CGA and should be obtained almost without exception in patients with suspected GCA on clinical grounds. It is essential because the treatment course for GCA is long and often
complicated, and many of the nonspecific symptoms of GCA such as headache, body aches, and fatigue may occur in many other diseases. A positive temporal artery biopsy is
100% specific but has a relatively low sensitivity of 15-40% for the diagnosis of GCA.

NOTE - Patients who only have the symptoms of polymyalgia rheumatica do not need a biopsy. However, such patients should be carefully questioned about headache, jaw or
arm claudication, visual symptoms, and any unusual pain in the face, throat, or tongue. Any of these symptoms suggest the possibility of GCA.

(Option B) ESR is performed to determine if prompt treatment with corticosteroids should be started. It is not diagnostic for CGA.

(Option C) Serologic tests, such as rheumatoid factor (RF), antinuclear antibodies (ANA), and cyclic citrullinated peptide (CCP) antibodies are typically negative, and not used for
diagnosis of CGA.

(Option D) Imaging studies such as CT scans are not routinely used to diagnose GCA unless another diagnosis is considered.

(Option E) Evoked visual potentials test is used for the diagnosis of optic neuritis and is not indicated here.

How well did you know this?
1
Not at all
2
3
4
5
Perfectly
21
Q

Mr. Johnson comes to your practice, complaining of progressively increasing left knee pain. He is 67 years old and otherwise healthy. He elaborates that the pain is brought on by walking. Generally, he feels well in the morning except for the stiffness of the left knee which often resolves within minutes of physical activity. When he starts walking, the pain is
brought on and builds up to a maximum intensity throughout the day. On physical examination, everything is normal except the crepitations of the left knee. The knee is not tender or warm to the touch and there is no swelling either. You order an X-ray of the left knee. Which one of the following would be the least likely finding on the X-ray?

A. Joint space narrowing.
B. Subchondral cysts.
C. Periarticular osteoporosis.
D. Osteophytes.
E. Increased density of bone ends.

A

Correct Answer Is C

This patient’s clinical presentation fits the diagnosis of osteoarthritis of the left knee. Osteoarthritis is the most common arthropathy worldwide.

The diagnosis of OA is mainly clinical. An X-ray of the affected joint is necessary to confirm the diagnosis. The characteristic findings on X-ray of an osteoarthritic joint are:
**Joint space narrowing
**Presence of subchondral cysts
**Sclerosis of subchondral bone
**Osteophytes at the outer edges of the joint lines

The pathophysiology of the disease includes chronic destruction of bone cartilage. With the diminished protective function of the cartilage, the periarticular bone undergoes repetitive trauma leading to bone destruction, which is followed by abnormal bone formation in form of bony spurs also called osteophytes.
Osteophyte formation plays a major role in the pathophysiology of pain in OA.

The healing process is affected by bony areas leads to sclerosis of the bony edges of the joint, signified by hyperdense sclerotic subchondral bones. Joint destruction and degeneration lead to joint space narrowing. Subchondral cyst formation would be another finding.

Osteopenia or osteoporosis around the joint is seen in inflammatory conditions such as rheumatoid arthritis or neuropathic arthropathy.

Neuropathic arthropathy (Charcot’s joint) may be seen in the following conditions:
**Diabetes mellitus neuropathy – is the most common cause in Australia and worldwide. It is related to long-term poor blood glucose control.
**Alcohol neuropathy
**Cerebral palsy
**Leprosy
**Syphilis (tabes dorsalis)
**Spinal cord injury
**Myelomeningocele
**Syringomyelia
**Intra-articular steroid injection
**Congenital insensitivity to pain
**Peroneal muscular atrophy

The classic findings on an X-ray of a joint with neuropathic arthropathy would be:
**Juxta-articular osteopenia
**Joint space narrowing

Osteopenia/osteoporosis around the joint (juxta-articular) is unlikely to be seen in OA.

How well did you know this?
1
Not at all
2
3
4
5
Perfectly
22
Q

A 17-year-old boy falls on his outstretched right arm, with subsequent shoulder pain and limited movement of his right shoulder, especially when attempting to elevate his arm over
the head. His appearance is illustrated in the following photograph. Which one of the following is the most likely diagnosis?

A. Fracture of the surgical neck of humerus.
B. Rupture of the rotator cuff tendon.
C. Dislocated glenohumeral joint.
D. Rupture of the long head of biceps tendon.
E. Dislocated acromioclavicular joint.

A

Correct Answer Is C

The flattened shoulder, as well as the way the patient is holding the affected arm to limit shoulder for preventing pain is typical for anterior glenohumeral dislocation.
Shoulder dislocations account for 50% of all major joint dislocations. In over 90% of cases, such dislocation occurs anteriorly. Posterior, inferior, or superior dislocations account for
the remaining 10%.

An anterior shoulder dislocation is usually caused by a blow to the abducted, externally rotated, and extended arm (e.g. blocking a basketball shot). Less commonly, a blow to the
posterior humerus or a fall on an outstretched arm may cause an anterior dislocation.
An anteriorly dislocated shoulder causes the arm to be slightly abducted and externally rotated. The patient resists all movement. There is loss of the normal round appearance of
the shoulder resulting in a flat shoulder as seen in the photograph.

(Option A) Fracture of the surgical neck of the humerus does not cause a flat shoulder. Pain and tenderness are almost always present. Bruising, swelling, or deformity may occur.

(Option B) Rotator cuff ruptures does not cause the shoulder to become flat. Pain and varying inability to abduct the arm are characteristic features.

(Option D) Rupture of the long head of the biceps tendon does not cause a flattened shoulder. Resisted flexion can elicit pain in the antecubital fossa. On forearm flexion, the bulk of the contracted biceps is lower than the unaffected arm.

(Option E) Acromioclavicular (AC) joint dislocation causes pain, tenderness and the classic step deformity in the AC joint. Arm raise is limited due to exquisite pain when attempted.

How well did you know this?
1
Not at all
2
3
4
5
Perfectly
23
Q

A young man presents to the emergency department after injuring his shoulder in a fall. His X-ray is shown in the following photograph. Testing the motor function by resisted
isometric contraction of which one of the following muscles would be most likely to confirm the presence of an associated nerve injury?

A. Latissimus dorsi.
B. Pectoralis major.
C. Trapezius.
D. Serratus anterior.
E. Deltoid

A

Correct Answer Is E

Significant findings on this X-ray of the shoudler are a humeral head out of the glenoid fossa and displaced inferiorly. These findings are typical for anterior dislocation of the
glenohumeral joint.

Shoulder dislocations often result in injury of the axillary nerve. Axillary nerve runs inferiorly to the humeral head and wraps around the surgical neck of the humerus. Axillary nerve
innervates the deltoid and terese minor muscles and the skin of the lateral shoulder (shoulder badge region). Axillary nerve dysfunction manifests as loss of sensation in a ‘shoulder badge’ distribution and deltoid muscle weakness. It is impractical to ask the patient to move the affected arm due to pain; however, in the presence of axillary nerve injury, the patient is often unable to isometrically contracts the deltoid muscle.
No other muscle mentioned in the options are innervated by the axillary nerve; hence, unaffected by axially nerve injury.

How well did you know this?
1
Not at all
2
3
4
5
Perfectly
24
Q

A patient with anterior dislocation of the shoulder is unable to perform which one of the following actions?

A. Turning the door knob.
B. Combing hair.
C. Taking a knife from the table.
D. Shaking hand.
E. Wrist extension.

A

Correct Answer Is B

Shoulder dislocation often result in injury of the axillary nerve. Axillary nerve runs inferiorly to the humeral head and wraps around the surgical neck of the humerus. Axillary nerve
innervates the deltoid and terese minor muscles and the skin of the lateral shoulder (shoulder badge region) Axillary nerve dysfunction manifests as loss of sensation in a ‘shoulder badge’ distribution and deltoid muscle weakness. Even in the absence of such injury, a patient with anterior shoulder dislocation cannot raise the arm; hence not able to comb hair.

Patients with anterior shoulder dislocation can use their elbow, wrists and fingers, so they can grasp and turn the door knob, pick and hold objects with their hand, shake hand or
extend the wrist.

How well did you know this?
1
Not at all
2
3
4
5
Perfectly
25
Q

Which one of the following are the most appropriate medications to give prior to reduction of an anterior dislocation of the shoulder?

A. IV diazepam and morphine.
B. IV midazolam.
C. Intranasal fentanyl and IV midazolam.
D. IV midazolam and morphine.
E. IM diazepam and nasal fentanyl.

A

Correct Answer Is C

Reduction of anterior shoulder dislocation is performed in the emergency department after adequate muscle relaxation and sedation is provided. Intravenous midazolam plus
intranasal fentanyl (an opiate) is the most commonly used drug combination.

Ketamine, etomidate and propofol are other appropriate alternatives.

Use of parenteral morphine is not justified while intranasal fentanyl can provide the same pain relief. Fentanyl is easier to administer. Diazepam is a long-acting benzodiazepine.
While a short duration of muscle relaxation is required for joint reduction, use of midazolam in preference of diazepam is more appropriate. Furthermore, IM diazepam has adelayed and unpredictable effect.

26
Q

After a fall on an outstretched hand, a 32-year-old woman develops pain and tenderness of her wrist. An X-ray is obtained that is shown in the following photograph. Which one of
the following is the most common complication of this fracture?

A. Joint stiffness.
B. Nonunion.
C. Misalignment.
D. Avascular necrosis.
E. Pain

A

Correct Answer Is D

The X-ray shows a scaphoid fracture of the left wrist. Scaphoid fracture is common and often results from fall on an outstretched hand. Tenderness of anatomical snuff box and
pain elicited by applying axial pressure to the thumb of the affected hand are typical clinical findings.

Avascular necrosis is the most common complication of scaphoid fracture seen in 13-40% of patients. Nonunion is the second most common complication occurring in 5-12%.
Joint stiffness and pain are other possible complications but are not as common.

Misalignment is not a complication of scaphoid fracture because the fracture is either non-displaced or displaced. Displaced fractures are treated with open reduction and internal
fixation. During the procedure the displacement is corrected and fixed.

27
Q

A 34-year-old woman trips over and falls on her outstretched right hand. Immediately after the fall she fells severe pain in her right wrist. On examination in the emergency
department, there is tenderness over the lateral aspect of the head of the radius and the anatomical snuffbox. Application of axial pressure to the thumb of the affected hand
causes pain. An X-ray is ordered which is completely normal. Which one of the following is the next best step in management?

A. Reassurance.
B. Place the wrist in thumb spica cast and repeat the X-ray in 7-10 days.
C. Repeat the X-ray in 6 weeks.
D. Analgesics.
E. Place the wrist in a sugar tongue splint

A

Correct Answer Is B

The mechanism of injury as well as the exam findings is suggestive of scaphoid fracture. Scaphoid fracture often results from falling on an outstretched hand. Tenderness of
anatomical snuff box and pain elicited by applying axial pressure to the thumb of the affected hand are typical clinical findings.
Once scaphoid fracture is suspected, plain X-ray films are the most appropriate next step in management. If fracture is evident on X-ray, definite treatment with casting or surgery to
fix a displaced fracture follows.
Some patients, however, will have normal X-rays, despite high index of clinical suspicion. In such cases, further management depends on the availability of advanced imaging studies such as MRI or CT scan. MRI is the preferred modality, as it is radiation-free and can assess both the scaphoid and the surrounding soft tissue. CT scan can detect fractures missed on plain radiographs but it cannot visualize possible soft tissue injuries.

If advanced imaging is not available, the wrist should be immobilized by a thumb spica cast for 7-10 days (up to 14 days) and X-ray repeated after this period. By 7-10 days, the
fracture line should be visualized on plain films. A negative control X-ray film in the presence of symptoms warrants referral for MRI.

(Option A) Scaphoid fracture may not be evident on initial X-rays, but seen on follow-up films. A negative X-ray in a symptomatic patient does not exclude fracture; so reassurance is
not appropriate.

(Options C and D) When symptoms suggest scaphoid fracture but X-rays are clear, the wrist should be immobilized and X-ray repeated in 7-10 days. Analgesic may be prescribed for
pain control, but giving only analgesics without wrist immobilization is incorrect.

(Option E) Sugar tongue splints are insufficient for immobilization of the wrist after scaphoid fracture. A thumb spica cast stabilizes the wrist in a neuter position with an slight ulnar
deviation.

28
Q

A 76-year-old lady presents to the emergency department after she sustained a fall on her outstretched hand. The X-ray obtained is shown in the following photograph. Which one of the following should be the position of the the wrist in cast?

A. Full flexion of the wrist with radial deviation.
B. Partial flexion of the wrist with radial deviation.
C. Full flexion of the wrist with ulnar deviation.
D. Partial extension of the wrist with radial deviation.
E. Partial flexion of the wrist with ulnar deviation.

A

Correct Answer Is E

The X-ray shows fracture of the distal radius, also known as Colles fracture. Falling on an outstretched (dorsiflexed) hand is the most common mechanism of this fracture. Colles
fracture is very common in elderly women, and can occur with minimal trauma if there is osteoporosis.

There can be posterior dislocation of the head of the radius, giving rise to the ‘fork deformity’. Lateral displacement may also occur. Casts are often used for treatment of Colles fractures. Severely displaced fractures may be treated with external or even internal fixation.

For casting, the wrist should be in 10° flexion and 10° ulnar deviation, and the forearm must be in slight pronation. Often a below-elbow cast for 4-6 week is sufficient; however, the cast may be extend to include above the elbow if there is significant displacement. Every significant displacement should be reduced under anesthesia before casting.

29
Q

A 72-year-old woman develops pain and deformity of the left wrist after she falls on her outstretched left hand. The x-ray is as shown in the following photograph. Which one of the
following is the characteristics of the cast that should be used for treatment?

A. Should include the wrist but not the thumb.
B. Should include the wrist and thumb.
C. Should include the wrist, thumb and elbow.
D. Should include the wrist and elbow.
E. Should include the thumb, wrist, elbow and the upper arm

A

Correct Answer Is A

The X-ray shows fracture of the distal 3cm of the radius, namely Colles fracture. Casting after meticulous reduction of any displacement remains the most commonly used treatment option for Colles fracture. The wrist is flexed at 10° and deviated to the ulnar at 10°. The cast includes the wrist and the forearm but not the thumb and the elbow.

However, if there is severe displacement and casting is considered for treatment, the cast may be extended to include up to above the elbow is selected patients.

30
Q

Which one of the following could be the earliest complication of the fracture shown in the following photograph?

A. Malunion.
B. Nonunion.
C. Ischemic Volkmann contracture.
D. Wrist drop.
E. Rupture of the extensor pollicis longus tendon.

A

Correct Answer Is C

The photograph shows fracture of the distal 3cm of the radius, referred to as Colles fracture. Potentially, Colles fracture can be associated with complications including:

**Rupture of extensor pollicis longus tendon
**Malunion and/or nonunion
**Stiffness of the elbow, metacarpophalangeal joints (MCPs), interphalangeal joints (IPs)
**Discomfort at distal radiolunalr joint
**Regional pain syndrome
**Volkmann contracture

Of these, Volkmann contracture develops earliest.

Volkmann ischemic contracture is the permanent shortening of forearm muscles, usually resulting from compartment syndrome in the forearm. It causes a claw-like deformity of
the wrist, hand, and fingers. The clinical presentation of Volkmann contracture includes five Ps: pain, pallor, pulselessness, paresthesia, and paralysis. Of these, pain is the earliest
sign.

On physical examination, pain accentuated by passive stretching seems to be the most reliable finding. Firmness of the tissues often is noted on palpation. Pulselessness and
paralysis are late findings. Induration of the forearm is another useful diagnostic finding.

NOTE - Any fracture of the humerus, elbow, forearm and wrist can result in compartment syndrome and Volkmann contracture.
Malunion (option A), non-union (option B) and rupture of the extensor pollicis longus tendon (option E) do not develop as early as Volkmann contracture. Rupture of extensor pollicis
longus tendon often occurs 4-8 weeks after the fracture. For wrist drop (option D), radial nerve injuries should happen at a more proximal level. Colles fracture does not cause wrist
drop.

31
Q

Which one of the following is the most common complication of Colles fracture?

A. Malunion.
B. Nonunion.
C. Ischemic Volkmann contracture.
D. Osteoarthritis of the wrist.
E. Wrist joint stiffness.

A

Correct Answer Is A

Colles’ fracture is defined as fracture of the distal 3cm of the radius.

The following are possible complications of Colles’ fracture:

* Rupture of extensor pollicis longus tendon
* Malunion and/or nonunion
* Stiffness of the elbow, metacarpophalangeal joints (MCPs), interphalangeal joints (IPs)
* Discomfort at distal radiolunalr joint
* Regional pain syndrome
* Volkmann contracture

Post-reduction redisplacement is the most common complication of Colles’ fracture that can result in malunion; therefore, the most likely complication among other options.

Nonunion (option B), wrist joint stiffness (option E), osteoarthritis of the wrist (option D), regional pain syndrome, median nerve injury and inferior radioulnar joint laxity are other
less common complications associated with Colles’ fracture. Volkmann’s contracture (option C) is a grave complication if compartment syndrome occurs in forearm fractures
(including distal radius fracture) but it is not as common.

32
Q

Which one of the hip joint movements is initially affected by osteoarthritis?

A. External rotation.
B. Internal rotation.
C. Flexion.
D. Adduction.
E. Abduction.

A

Correct Answer Is B

In osteoarthritis of the hip joint (and other hip joint pathologies), the first movement restricted is internal rotation of the hip followed by hip flexion.

33
Q

A 48-year-old man presents to the Emergency Department with complaint of sudden onset pain and tenderness that developed after he lifted an armchair at home for redecoration.
One examination, restricted and painful movements of the lumbar area including flexion, extension and rotation, as well as tenderness over the L4-L5 region is noted. Which one of the following is the most appropriate step in initial management of this patient?

A. Arrange for an MRI scan of the lumbosacral spine.
B. Prescribe lumbar corset.
C. Prescribe analgesics and advise that he continue his daily activities.
D. Prescribe analgesics and advise bed rest.
E. Prescribe pelvic traction.

A

Correct Answer Is C

This scenario represents a case of acute low back pain. Low back pain is a very common condition faced in general practice. It is estimated that up to 80% of adults have low back pain at some times in their lives. Despite the fact that there is a long list of potential causes for low back pain, most patients seen in primary care (>85%) will have nonspecific low
back pain meaning that there is back pain in the absence of a specific underlying condition that can be reliably identified. Only less than 1% are found to have a serious underlying disease such as cancer or infection.

In the middle of this spectrum, there are other less serious but specific etiologies, the most important of which are vertebral compression fractures, radiculopathies, and spinal canal
stenosis. Other conditions include osteoarthritis of the lumbosacral spinal column, scoliosis and hyperkyphosis, psychologic stress and extraspinal causes such as pancreatitis,
nephrolithiasis, pyelonephritis, abdominal aortic aneurysms and herpes zoster.

In approaching patients with low back pain, the first step in management is making a judicious decision as to which patient can be managed conservatively versus those who may need more investigations and probably referral for specialist care. As always, a thorough history and physical examination is the most important step in approaching patients with low back pain. The very first thing to consider in history and during physical exam is to assess the presence of alarming features or risk factors that may point toward a serious underlying condition.
Such features and risk factors are known as ‘red flags’ and are summarized in the following table: (For clearer table, please see page 1168-1169)

Red flags should be evaluated in the context of the clinical presentation as a whole, not as the sole guide for management because over 80% of patients with non-specific low back
pain have a red flag in their histories irrelevant to their back pain. High index of suspicion should be kept when there is more than one red flag present.
Physical examination of patients with low back pain includes inspection for any deformities in the spinal column, hip and lower limb such as scoliosis, kyphosis, skin changes, signs
of trauma like bruises or abrasions, and hip and lower limb deformities or discrepancies. Checking the range of motion of lumbar or lumbosacral spine should be performed in all
patients if possible, as they might give clues about the extent of disability and, at occasions, the probable source of the pain (e.g., a restricted flexion in disc problems versus a
limited extension in facet-joint disorders).
Palpation of the spine and paraspinal areas is another part of the physical exam. It is important to note that local tenderness in the lumbar area is a common finding in non-specific
low back pain. This tenderness is caused mostly by the spasm of the paravertebral muscles and is not an alerting sign (red flag). For example, a tear in a facet joint or a prolapsed or
degenerated disc can result in muscle spasm leading to tenderness over the underlying area. However, bone tenderness directly over lumbar spinous processes should be
considered as an alarming sign pointing toward serious bone disorders such as neoplasm or infection.
Another important point to consider is that as long as the ow back pain remains above the sacroiliac joint, a neurological exam is not required.

As a rule, all patients with low back pain without a red flag should be initially managed conservatively with:
1. Education and explanations
2. Simple analgesics such as paracetamol or NSAIDs for pain control
3. Encouraging maintenance of daily life and remaining physically active
4. Recommending and referral for physiotherapy for those with debilitating pain and/or severe restriction of movements
5. Review and reassessment (recommended in one and 4-6 weeks)

It is very important to note that NO further investigation including imaging studies such as plain radiographs, CT scan or MRI are indicated in patients with low back pain in the
absence of red flags. Such studies should be considered if a red flag is present or the symptoms persist despite effective conservative management (as above) for at least 4 weeks.
This holds true for patients with radicular pain (e.g. sciatica); however, any patient with symptoms of cauda equina, spinal cord compression, or progressive and/or severe
neurologic deficit should have immediate MRI for evaluation as well as urgent specialist referral. Symptoms indicating these conditions include: new bowel or bladder incontinence,
urinary retention, saddle anesthesia and significant motor deficit not localized to a single unilateral nerve root. This means that, for example, a patient with low back pain, unilateral
foot drop and no red flags does not need further investigations until conservative management proves ineffective after 6 weeks of treatment,
For patients with a red flag further investigation and/or referral may be indicated. The investigations of choice differ based on the probability diagnoses formed through history and
physical findings. Despite being non-sensitive and non-specific, plain radiography is often the initial imaging study. Detailed and more specific investigations are then considered
based on the probability diagnosis.
This man has low back pain with typical findings of painfully restricted movements and tender L4-L5. Importantly, he has no red flags in the history and physical examination;
therefore, a diagnosis of non-specific low back pain can be considered for now. Of the options, analgesia and maintenance of physical activity is the best initial management for
him. Physiotherapy can be discussed and offered for symptom relief thereafter.

(Option A) Imaging studies such as plain radiography, CT scan, or MRI are not indicated in evaluation of non-specific low back pain, unless there is at least one red flag present.
Choice of further investigation depends on other pieces of information from history and physical examination.

(Option B) Although widely used, several studies found little or no difference between individuals with low-back pain who used lumbosacral corsets as a means of back support and
those who received no treatment.

(Option D) Patients with non-specific low back pain should be encouraged to continue their daily routines and remain physically active. Bed rest should be strongly discouraged.

(Option E) Pelvic traction is an irrelevant and inappropriate option for treatment of non-specific low back pain.

34
Q

A 47-year-old woman presents to your practice with complaint of progressive right elbow pain for the past two months. She is an office secretary and her job entails lots of computer
keyboard and mouse use. The pain increases through the day as she performs her tasks. Recently, the pain is also brought up by gripping, and sometimes persists at night. On
examination, there is no deformity, redness or warmth of the right elbow, and the range of motion is normal. However, palpation elicits tenderness approximately 2 cm below the
right lateral epicondyle. Fingers and wrist have normal range of motion but passive terminal flexion of the wrist and resisted extension of the wrist and middle finger reproduce the
pain. Which one of the following is the most likely cause of this presentation?

A. Olecranon bursitis.
B. Lateral epicondylitis.
C. Medial epicondylitis.
D. Rupture of the wrist extensor tendons.
E. Stress fracture of the right epicondyle.

A

Correct Answer Is B

The scenario represents a typical case of humeral lateral epicondylitis, also known as lateral tennis elbow. Lateral epicondylitis is the most common overuse syndrome of the upper
limb and is related to excessive wrist extension.
The typical age of onset is 40 to 50 years. There is almost always a history of overuse without specific trauma.
Lateral epicondylitis presents with pain in the lateral elbow that worsens with activity and alleviates with rest. The pain often can be pinpointed 1-2 cm distal to the proximal origin of the extensor carpi radialis brevis muscle (ECRB) at the lateral epicondyle. Affected individuals commonly describe aggravating activities such as a backhand stroke in tennis or the overuse of a screwdriver, computer keyboard or mouse. Pain can vary from being mild (e.g. with aggravating activities like tennis or the repeated use of a hand tool), or it can be
such severe that simple activities like gripping of even light objects such as coffee cup (i.e. “coffee cup sign”) are painful. The pain may also radiate down the posterior aspect of the forearm.

On examination, the elbow has normal appearance and range of motion. However, pronation at its extreme can be painful. Resisted extension of the wrist and the middle finger as well as the passive terminal flexion of the wrist while the elbow is fully extended reproduce the pain. It is important to notice that lack of such findings does not exclude lateral epicondylitis as a likely diagnosis.

Lateral epicondylitis is a benign self-limiting condition that often resolves within 6 to 12 months in most cases. Activity modification, counterforce bracing, and home-based exercise
programs constitute the initial conservative management in almost all affected persons.

(Option A) Olecranon bursa is a subcutaneous synovial-lined sac overlying the olecranon process at the proximal end of the ulna. This bursa cushions the olecranon and decreases
friction between it and the skin. Olecranon bursitis is the inflammation of this bursa that most commonly presents with a focal swelling at the posterior elbow. Pain may or may not
be reported by the patient but is often absent in noninflammatory aseptic cases. If present, pain is usually exacerbated by pressure, such as when the patient leans on the elbow or
the elbow is rubbed against a surface like tabletop during writing. Many cases are idiopathic. Infection is a less common but important cause occasionally (septic bursitis).

(Option C) Medial epicondylitis is a condition similar to lateral epicondylitis in pathophysiology. It affects the tendons of wrist flexor muscles the pronator teres and flexor carpi radialis at their proximal insertion at the humeral medical epicondyle. Overuse of the wrist flexor muscles by excessive wrist flexion is the main etiology. The pain is felt in the medial
aspect of the forearm and can be reproduced by resisted flexion of the wrist or passive terminal wrist extension with the elbow in full flexion. There is often tenderness over the
medial epicondyle and the proximal wrist flexor muscles mass. The ulnar groove, ulnar nerve and medial collateral ligament should NOT be tender. Medial epicondylitis is managed
similar to lateral epicondylitis with activity modification, rest from the causative activity, home-based exercises to strengthen the wrist flexor muscles, ice, NSAIDs, or physiotherapy.

(Option D) With the rupture of the wrist extensor muscles tendons, wrist extension was expected to be defected which is not the case here.

(Option E) Although the stress fracture of the right lateral epicondyle can present with an almost similar presentation (expect for the maximal tenderness being elicited over the
lateral epicondyle itself rather than 1-2 cm below it), it is not a likely diagnosis because it is a very rare condition that often occurs in children. Stress fractures of the medial
epicondyles are much more common than those of right epicondyle.

35
Q

Which one of the following wrist movements is most likely to reproduce the pain in a patient with lateral epicondylitis?

A. Radial deviation.
B. Resisted extension.
C. Supination.
D. Ulnar deviation.
E. Resisted flexion.

A

Correct Answer Is B

Lateral epicondylitis, also referred to as ‘lateral tennis elbow’ is the most common overuse syndrome related to excessive wrist extension. Patients with lateral epicondylitis typically
present with pain in the lateral aspect of the forearm that may radiate down into the forearm. There is often maximal tenderness 1-2 cm distal to the origin of the extensor carpi
radialis brevis muscle (ECRB) at the lateral epicondyle.

The pain in lateral epicondylitis is reproduced with resisted wrist extension with the elbow fully extended, or passive terminal wrist extension while the elbow is fully extended.
Resisted extension of the middle finger is also painful secondary to stress placed on the ECRB tendon, as it is preferentially stressed in this position when it must contract
synergistically to anchor the third metacarpal, such that extension can take place at the digits. Resisted pronation of the wrist and gripping while the elbow is extended can also
bring up the pain.
Of the given options, only resisted extension of the wrist can reproduce the pain in a patient with lateral epicondylitis.

(Options A and D) Radial and ulnar deviation of the wrist do not exert a significant force on extensor muscles and are unlikely to reproduce the pain.

(Option C) supination of the wrist, especially if resisted, can reproduce the pain in medial epicondylitis (golf elbow), as does the resisted flexion of the wrist (option E)

36
Q

Overuse of which one the following is most likely the underlying mechanism of lateral epicondylitis?

A. Wrist extension.
B. Wrist flexion.
C. Elbow flexion.
D. Elbow extension.
E. Wrist pronation.

A

Correct Answer Is A

Lateral epicondylitis, also referred to as ‘lateral tennis elbow’ is the most common overuse syndrome related to excessive wrist extension.

The lateral epicondyle of the humerus serves as the bony common origin of the wrist extensors. Overuse injury to the extensor carpi radialis brevis muscle (ECRB), which is felt at the tip of lateral epicondyle and occasionally, the extensor digitorum communis muscle (EDC) (felt just posterior and lateral to the tip of lateral epicondyle) constitutes lateral epicondylitis.

‘Tennis elbow’ and ‘lateral epicondylitis’ are, however, misnomers because the condition is seen more commonly in non-tennis players than tennis players and the underlying
pathophysiology is degenerative rather than inflammatory. Microscopic evaluation of the tendons does not show signs of inflammation, but rather angiofibroblastic degeneration
and collagen disarray. Light microscopy reveals both an excess of fibroblasts and blood vessels that are consistent with neovessels or angiogenesis. The injury to the tendon often
occurs 1 to 2 cm below the attachment point of ECRB to lateral epicondyle.

37
Q

A 45-year-old man presents to your GP clinic with complaint of a painful right elbow. The pain is felt laterally radiating to the back of the forearm, aggravates with activity and
relieves with rest. He is a carpenter and makes cabinets. His job includes lots of sawing, drilling, and using screw drivers. On examination, you notice a tender point 1-2 cm distal to
the right lateral epicondyle. Extension of the wrist with the elbow in full extension brings up the pain. Which one of the following is the most appropriate next step in management?

A. Physiotherapy.
B. Advising him to quit his job.
C. Intralesional corticosteroids.
D. Surgery.
E. Band support below the elbow.

A

Correct Answer Is E

The scenario describes a typical case of lateral epicondylitis, also known as ‘lateral tennis elbow’, which is the most common overuse syndrome of the upper limb and is related to
excessive wrist extension. The condition is typically seen in persons of 40-50 years of age.
Affected individuals often report an insidious onset, with a history of overuse without specific trauma almost always present in the history. Patients with the condition complain of
pain over the lateral elbow that worsens with activity and alleviates with rest. The pain often can be pinpointed 1-2 cm distal to attachment of the extensor carpi radialis brevis
muscle (ECRB) tendon to the lateral epicondyle.
Lateral epicondylitis is benign self-limiting condition that often resolves within 6 to 12 months in most cases. Activity modification, counterforce bracing, and home-based exercise
programs constitute the initial conservative management in almost all cases. This patient should be advised to modify all his daily activities in a way that limits forearm pronation,
the position in which wrist extension exerts great force on wrist flexor tendons. This often includes using a palm-up position to avoid or limit pronation.

Of the given options band support below the elbow (counterforce bracing) is the most appropriate option to consider for him. Counterforce braces encircle the forearm 6 to 10 cm to reduce the tension forces on the wrist extensor tendons and provide rest for the extensor tendons. Despite conflicting evidence, they are still widely used in management of lateral
epicondylitis. Counterforce braces are inexpensive, convenient, and have shown to decrease pain and increase grip strength in most cases within 3-6 weeks.

Application of ice and short-term use of topical or oral non-steroidal anti-inflammatory drugs (NSAIDs) are other measures that might be considered for acute management of
lateral epicondylitis.

(Option A) Some studies have shown that physiotherapy can be more effective than a wait-and-watch approach or topical corticosteroid injection in both the short and long-term outcomes; however, it is often reserved for those who fail to respond to the above measures such as home-based exercise programs.

(Option B) Although this patient has developed lateral epicondylitis most likely due to the overuse of his elbow in his carrier, advising him to quit the job he is doing for a living is not
appropriate and practical.

(Options C) Intralesional corticosteroid injection is reserved for patients with refractory severe pain and disability despite conservative management. Corticosteroid injection is
associated with a rapid response in short term (usually within 4 weeks) but poorer outcome in long term.

(Option D) Surgery is rarely used for treatment of patients who fail to response to all available measure for treatment of lateral epicondylitis.

38
Q

Janet is a 43-year-old IT worker, who has presented to you with complaint of pain in the right elbow. She describes that the pain increases through the day as she does her job which includes lots of typing with computer keyboard and use of computer mouse. On examinations, the elbow looks normal with preserved range of motion, but slight pain at the extreme of pronation. Tenderness is elicited over the lateral epicondyle of the right humerus when palpated deeply. The pain is reproduced on resisted extension of the wrist while her forearm is fully extended. The arm is otherwise normal. Which one of the following would be the best treatment option for Janet at this stage?

A. Immobilization of the elbow.
B. Corticosteroid injection.
C. Encircling support strap for the right forearm.
D. Oral NSAIDs.
E. Topical NSAIDs.

A

Correct Answer Is C

The scenario is a typical description of lateral epicondylitis, which is caused by degenerative changes of the wrist extensor muscles at their origin in the humeral lateral epicondyle.
Of these muscles, the extensor carpi radialis brevis muscle (ECRB) is the most commonly affected one.

The diagnosis is almost always clinical, but x-rays, ultrasound, or MRI can be used when the diagnosis is in doubt or for further management planning in refractory cases.
A myriad of treatment options has been investigated and applied for treatment of lateral epicondylitis; however, conservative management including activity modification to prevent further tendon overuse, counterforce bracing, and home-based exercises to increase the strength of the extensor muscles is almost always the best initial option to consider.

Although controversial, short term use of topical (or sometimes oral) non-steroidal anti-inflammatory drugs (NSAIDs) may be prescribed for acute and very painful cases.

Application of ice can also help with pain reduction in an acute phase. The outcome is then reviewed, often in 4 to 6 or 8 weeks for further management.

Patients with lateral epicondylitis should be advised and taught to modify their arm movements in a way that most of their activities are done with the palm facing up rather down in an attempt to prevent or limit forearm pronation in which wrist extension can exert excess force on the wrist extensor tendons. However, for some patients, whose daily activity, including their job, entails tasks such as typing or computer mouse clicking, such instruction may be impractical to follow.

Another important step in management is counterforce bracing of the forearm by placing an encircling band around it. Counterforce braces are strapped around the forearm below
the elbow to reduce the tension forces on the wrist extensor tendons and provide rest, an important part of the initial management, for the extensor tendons. Despite conflicting
evidence, counterforce braces are still widely used. Counterforce braces are cost-effective and easy to use, and may provide benefit during the first 6 weeks of treatment by pain relief and grip strengthening. An encircling support strap below the elbow acts as a counterforce brace for Janet and is the best choice among the given options.

(Option A) Immobilization of the elbow has been shown to be inferior to counterforce bracing below the elbow in pain reduction and increasing grip strength.

(Option B) Intralesional corticosteroid injection is reserved for patients with refractory severe pain and disability despite conservative management. Corticosteroid injection is
associated with a rapid response in short term but poorer outcome in long term.

(Options D and E) Lateral epicondylitis is a tendinosis rather than tendinitis with minimum or absent inflammatory changes on histopathology. Despite the absence of apparent inflammation, NSAIDs either topical or oral (if not contraindicated) have shown benefit in short term (up to 14 days) in some studies. Although controversial, they are often used for
patients with lateral epicondylitis, specially during the acute initial phase. Diclofenac is the preferred NSAIDs as it has showed efficacy in some studies (to date naproxen has proved
not more effective than placebo).

39
Q

A 48-year-old man presents to your practice, complaining of the soreness of his right elbow. He is an office worker who uses the computer keyboard and mouse a lot at work, to
which he thinks his pain is related. On examination, the right elbow has both normal appearance and range of motion. There is, however, tenderness over the lateral epicondyle. Wrist extension while the forearm is extended reproduces the pain. Which one of the following is the most appropriate treatment option for him?

A. Finger immobilization splint.
B. Rest and paracetamol.
C. Immobilization of the elbow.
D. Using encircling support orthotic on forearm.
E. Topical diclofenac

A

Correct Answer Is D

This man has presented with a clinical presentation that is very typical for lateral epicondylitis. Lateral epicondylitis is caused by degenerative changes of the wrist extensor muscles at their origin in the humeral lateral epicondyle. The extensor carpi radialis brevis muscle (ECRB) is the most commonly affected wrist extensor muscle.

For patients with lateral epicondylitis, conservative management with activity modification to prevent further tendon overuse, counterforce bracing, and home-based exercises to strengthen the wrist extensor muscles is almost always the best initial management option.

Persons with lateral epicondylitis should be instructed to modify their wrist and elbow movements in a way and direction that makes the palm facing up rather than down to prevent
or limit forearm pronation in which wrist extension can exert excess force on the wrist extensor tendons. This, however, is not applicable for all patients because their daily activity, including their job, entails activities such as typing or computer mouse clicking.

Another important step in management is counterforce bracing of the forearm by placing an encircling band around it. Counterforce braces are wrapped around the forearm below
the elbow and reduce the tension forces on the wrist extensor tendons and provide rest, which is a very important portion of initial conservative management. Although evidence is
conflicting and controversial, counterforce braces are still widely used and deemed useful for treatment because they are cheap and easy to use and may provide benefit during the
first 6 weeks of treatment. An encircling support strap below the elbow acts as a counterforce brace for this patient and the best choice among other options.

(Option A) Fingers extension does not exert force on wrist extensor tendons; therefore, finger immobilization is not effective in providing rest for the wrist extensor tendons at the elbow.

(Option B) Use of paracetamol as a non-prescribed analgesic has been used for alleviation of pain in short term. Advice for rest, however, is not practical in this scenario as the
nature of his job entails use of computer keyboard and mouse, during which the causative wrist and forearm movements are continued. For him, counterforce strapping decreased tension on wrist extensor tendons while he can continue his daily activities.

(Option C) Immobilization of the elbow has been shown to be inferior to counterforce bracing below the elbow in pain reduction and increasing strength of grip.

(Option E) Although controversial, short term use of topical (or sometimes oral) non-steroidal anti-inflammatory drugs (NSAIDs) may be considered for acute and very painful cases.
However, rest from the implicated activity by means of counterforce bracing remains the most appropriate initial option.

40
Q

Which one of the following is not considered an extra-articular manifestation of rheumatoid arthritis?

A. Hepatomegaly.
B. Osteoporosis.
C. Pericardial effusion.
D. Cutaneous nodules.
E. Peripheral neuropathy

A

Correct Answer Is A

Extra-articular RA is a serious condition, and RA patients with extra-articular manifestations should be treated and monitored aggressively. Extra-articular manifestations of RA occur
in approximately 40% of patients, either in the beginning or during the course of their disease. Extra-articular manifestations of RA include the following:

Bone manifestations:

Osteopenia and osteoporosis are very common in patients with RA. The development of osteopenia in RA patients appears to occur independently of corticosteroid use

Skin manifestations:
**Rheumatoid nodules (the most frequent skin manifestation seen in 20% of patients with RA)
**Pyoderma gangrenosum

Ocular manifestations:
**Keratoconjunctivitis sicca – affects at least 10% of RA patients. It is frequently observed together with xerostomia in a secondary Sjögren syndrome.
**Episcleritis (in 1% of RA patients)
**Scleritis
**Peripheral ulcerative keratitis

Oral manifestations:
**Oral dryness (xerostomia) and salivary gland swelling can be found in patients with RA. These patients can also develop secondary Sjögren syndrome.

Gastrointestinal manifestations:
**Gastrointestinal manifestations of RA are more related to treatment rather than the disease. A rare gastrointestinal problem seen in RA patients is caused by mesenteric
vasculitis leading to intestinal infarctions, bleeding, and perforation.

Pulmonary manifestations:
**Pleural disease (in 50% of RA patients)
**Parenchymal pulmonary nodules
**Interstitial lung disease

Cardiac manifestations:
**Pericarditis (the most common cardiac manifestation of RA)
**Myocardial infarction
**Myocarditis
**Congestive heart failure
**Endocarditis

Renal manifestations:
Renal manifestations of RA are rare and include:
**Mesangial glomerulonephritis
**Amyloidosis

Neurological manifestations:
**Peripheral neuropathy occurs in a small subset of patients
**Cervical myelopathy caused atlantoaxial subluxation or pannus formation.

Hematological manifestations:
**Anemia (one of the most common extra-articular manifestations of RA)
**Neutropenia
**Thrombocytosis
**Lymphadenopathy

Of the given options hepatomegaly is not an extra-articular manifestation of RA.

41
Q

While getting off the car, a 40-year-old man developed sudden-onset severe low back pain and got locked in the middle of the way, according to him. He is now in the emergency
department for evaluation, complaining of lower back pain, shooting sharp pain in the right leg and foot, and restricted and painful lumbar movements. He runs a grocery store and has been otherwise fit. He drinks alcohol on social occasions and smokes 3 to 4 cigarettes a day. His past medical history is insignificant. On examination, he has stable vital signs and tenderness over the L4-L5 region. Except for positive slump test and straight leg raise test, neurological exam findings are inconclusive with no motor or sensory deficits. Which one of the following is the most appropriate management option for him?

A. Obtaining a lumbosacral spine x-ray.
B. Obtaining a lumbosacral spine CT scan.
C. Obtaining a lumbosacral spine MRI.
D. Nerve conduction studies.
E. Observation.

A

Correct Answer Is E

It is estimated that up to 80% of adults have low back pain at some times in their lives. Despite the fact that there is a long list of potential causes for low back pain, most patients
seen in primary care (>85%) will have nonspecific low back pain, meaning that there is back pain in the absence of a specific underlying condition that can be reliably identified. Only less than 1% are found to have a serious underlying disease such as cancer or infection.

In the middle of this spectrum, there are other less serious but specific etiologies, the most important of which are vertebral compression fractures, radiculopathies, and spinal canal
stenosis. Other conditions include osteoarthritis of the lumbosacral spinal column, scoliosis and hyperkyphosis, psychological stress and extraspinal causes such as pancreatitis,
nephrolithiasis, pyelonephritis, abdominal aortic aneurysm and herpes zoster.
In this patient, the acute onset of back pain shooting down the leg and foot and positive slump and straight leg tests are highly suggestive of radicular pain (sciatica). Moreover, he has no ‘red flag’ (see ‘NOTE’ below).

Current guidelines recommend that patients with radicular pain, who have no red flag, be managed conservatively with:
1. Education and explanations
2. Simple analgesics such as paracetamol or NSAIDs for pain control
3. Encouraging maintenance of daily life and remaining physically active
4. Recommending and referral for physiotherapy for those with debilitating pain and/or severe restriction of movements
5. Review and reassessment (recommended in one and 4-6 weeks)
6.
This means that patient with radicular pain do not require any studies such as plain X-rays (option A), CT scan (option B), MRI (option C), or nerve conduction studies (option D). An
exception is the presence of significant motor deficit not localized to a single unilateral nerve root. In such patients urgent MRI and referral is the management of choice. This
means that, for example, a patient with low back pain, unilateral foot drop and no red flags does not need further investigations until conservative management proves ineffective
after six weeks of treatment.

Of the options, observation while the patient is on conservative management as mentioned above is the most appropriate next step in management.

NOTE - Red flags for low back pain are summarized in the following table:

42
Q

A 55-year-old man presents with sudden onset severe pain over the L4/L5 region which is sharp and shooting in nature and radiated down to both legs. He has never had such pain
before and there is no history of preceding trauma or strain. He has lost six kilograms in the past two months. Which one of the following in the history is the most important factor necessitating further investigations in this patient?

A. The first episode ever.
B. No history of preceding trauma or strain.
C. His age.
D. Weight loss.
E. Radicular nature of his pain

A

Correct Answer Is D

Despite the long list of conditions resulting in low back pain, most patients seen in primary care (>85%) will have nonspecific low back pain meaning that the patient has back pain in
the absence of specific underlying condition that can be reliably identified. Of those who present to primary care, only less than 1% are found to have a serious underlying disease
such as cancer or infection.

In the middle of this spectrum, there are other less serious but specific etiologies, the most important of which are vertebral compression fractures, radiculopathies, and spinal canal
stenosis, etc.

In management of patients with low back pain, the first step is making a judicious decision as to which patient can be managed conservatively versus those who may need more
investigations and probably referral for specialist care. Such assessment starts with history taking and physical examination and checking for the presence of any alarming feature or risk factors that may point toward a serious underlying condition.

Such features and risk factors are known as ‘red flags’ and are summarized in the following table: (see photo)

The presence of red flags is an indication for further evaluation and referral for further specialized management. One important issue is that red flags should be evaluated in the
context of the clinical presentation as a whole, not as the sole guide for management because over 80% of patients with non-specific low back pain have a red flag in their histories
irrelevant to their back pain. High index of suspicion should be raised when there is more than one red flag present.

Of the options, the patient’s age (>50 years) and the recent history of weight loss are alarming features with the latter being of significant importance. Back pain in the presence of a
history of recent unintentional weight loss, especially in someome older than 50 years, should always raise suspicion about malignancies for which meticulous assessment is prompted.

Being the first episode ever (option A) is not a red flag, neither is absence of a preceding trauma or sprain (option B). In fact, in most patients with low back pain no precipitating
event is identified.

(Option C) Age older than 50 years or younger than 20 years is a red flag. The age alone, however, is not as significant as the recent weight loss in this man.

(Option E) Radicular pain is quite common. Radiculopathies are often benign and managed conservatively with resumed daily activities, analgesics, and in more severe cases
physiotherapy. Investigations such as imaging or electrophysiological studies are unnecessary unless there is significant motor deficits not justified by a single nerve root
involvement, suspicion against cord compression, or cauda equina syndrome, for which urgent MRI is mandatory.)

43
Q

A 72-year-old patient of yours has presented to your clinic for advice regarding osteoarthritis of his right knee. He finds walking difficult due to pain. You recommend that he use a
stick as a walking aid. He needs to know how he should use it. Which one of the following is the appropriate recommendation for holding the stick and walking?

A. He should hold the stick in his right hand and step with the right leg first.
B. He should hold the stick in his right hand and step with the left leg first.
C. He should hold the stick in his left hand and step with the right leg first.
D. He should hold the stick in his left hand and step with the left leg first.
E. He should hold the stick in his right hand and can step with either leg

A

Correct Answer Is C

When using a stick as a walking aid for problems of a lower limb (foot, ankle, knee, or hip), the stick should be held with the hand on the unaffected side (good leg) and the first step
should be taken with the affected limb (bad leg).

As this patient’s problem is in his right leg, he should hold the stick with his left hand (the side of the good leg) and put his right foot
(bad leg) forward first.

When walking up stairs the good leg should take the first step and when walking down take the first step with the injured or affected leg.

44
Q

While gardening, a 45-year-old woman develops sudden onset low back pain. She is in the Emergency Department for evaluation now. Her past medical history is significant for
breast cancer 10 years ago that was treated with breast-preserving surgery, adjuvant chemotherapy and radiotherapy. She has been disease-free since then. On examination, there istenderness over the L4/L5 region, and painfully restricted flexion, extension and rotation. Neurological exam is unremarkable. Which one of the following is the most appropriate next step in management?

A. Physiotherapy.
B. MRI of the lumbosacral spine.
C. X-ray of the lumbosacral spine.
D. Advise that she continue daily activities.
E. Advise strict bed rest for 48 hours and analgesics.

A

Correct Answer Is C

Acute episodes of low back pain are very common and often managed conservatively if a thorough history and physical examination is negative for alarming signs, symptoms, and
risk factors necessitating prompt investigations for a serious underlying cause. Such alarming features are termed ‘red flags’ for low back pain and are summarized in the following
table:

With the presence of an active, treated or currently-in-remission malignancy in history, the back pain should be considered to have been caused by bone metastasis until proven
otherwise.

The bone is one of the most common sites of metastasis. A history of cancer, except non-melanoma skin cancers, is a very serious alarming factor for back pain from bone
metastasis. Breast, prostate, lung, thyroid, and kidney cancers account for 80% of metastases. Almost 60% of patients with multiple myeloma have skeletal lytic lesions present at
diagnosis.

Pain is the most common symptom in of metastatic bone disease. In patients with history of cancer, sudden, severe pain raises concern for pathologic fracture. Neurologic
symptoms from spinal cord compression or spinal instability could be another manifestation.

Where a pathologic fracture is suspected, plain radiography is often the initial imaging study. Detailed and more specific investigations are then considered based on the probability
diagnosis. For this woman, an X-ray of the lumbosacral spine to look for possible vertebral fracture(s) due to metastatic disease is the most appropriate initial step in management
among other options.

Conservative management with advice of continued activity (option D), short-term use of analgesics, or physiotherapy (option A) if the pain is severe and disabling are steps to
consider once a serious underlying pathology for the back pain has been safely excluded.

(Option B) MRI is often indicated in patients with radicular pain that cannot be solely attributed to one nerve root. Generally, radiculopathies are managed conservatively with analgesia, physiotherapy and encouraging maintained physical activity.

(Option E) Although simple analgesics or a short course of NSAIDs are often advised for management of nonspecific or benign low back pain, strict bed rest should be discouraged.
Patients should return and maintain daily physical activity as soon as possible.

45
Q

A 58-year-old woman presents to your GP clinic with complaint of back pain that started suddenly after one hour of gardening. Her past medical history is unremarkable. She has
been healthy and active until the event. She does not smoke and drinks alcohol in moderation. On examination, flexion, extension and rotation of the lumbar spine are significantly limited and tenderness between L4 and L5 is elicited. There is no tenderness over the vertebral bodies. After prescribing analgesics, which one of the following would be the most appropriate next step in management?

A. Bed rest.
B. Referral for orthopedic consultation.
C. Use of corsets.
D. Advise that she should maintain physical activity.
E. Magnetic resonance imaging of the lumbar spine.

A

Correct Answer Is D

Low back pain in the absence of alarming features is history and physical examination does not require specific management or evaluation other than reassurance, advising
maintained physical activity, short-term use of analgesics and review in 4 to 6 weeks.
Physiotherapy may be considered for patients with protracted severe pain despite initial
measures.

Alarming features are a set of signs, symptoms, or specific risk factors, the presence of which raise concerns regarding a serious underlying etiology for the low back pain such as
malignancies, infections, cord compression, cauda equina syndrome, etc. This occurs in approximately 1% of patients presenting with acute low back pain.

Alarming features, also known as ‘red flags’ are summarized in the following table: (see photo)

Red flags should be evaluated in the context of the clinical presentation as a whole, not as the sole guide for management, because over 80% of patients with non-specific low back
pain have a red flag in their histories irrelevant to their back pain. High index of suspicion is raised when there is more than one red flag present.

With the age older than 50 years, this woman has a red flag. Although low back pain in patients older than 50 can have been caused by a pathologic fracture due to metastasis or
osteoporosis, negative findings in history and physical findings make such diagnoses weak possibilities. The age alone in this context is not a red flag justifying further assessment;
imaging studies in particular.

NOTE - There are often abnormal findings on imaging studies of adults without low back pain which can make it difficult to correlate symptoms with imaging findings in
patients with low back pain. Disc herniation is seen on MRI scans of 22 to 67% of asymptomatic adults and spinal stenosis in 21% of adults older than 60 years. Evidence of
osteoarthritis is often seen on imaging but correlates poorly with symptoms.

Even when the radiographic findings are consistent with clinical presentation, the magnitude of radiographic findings does not necessarily correlate with clinical severity and
outcome, and clinical improvement may not correlate with resolution of the radiographic defect.

For this woman, advising maintained physical activity is the best management option. She can take analgesics if the pain is bothersome. A review in 4 to 6 weeks should follow to
see the response to treatment and further management planning.

(Option A) Bed rest should be discourage because immobility is associated with poorer outcomes.

(Option B) Referral for orthopedic or neurological consultation could be a correct option if there was any pointers towards a more serious cause other than non-specific low back
pain.

(Option C) Corsets provide support and correction of posture and have shown benefit in some studies on patients with chronic low back pain due to anatomical and structural
deformities. Their use in patients with acute low back pain has not been associated with significant improved outcomes. Therefore, they are not recommended for management of
acute low back pain.

(Option E) MRI would be the option of choice if there was any significant motor deficit not justified by a single nerve root pathology, any suspicion of cord compression, or cauda
equina syndrome. This woman does not have any clinical features suggestive of these.

46
Q

Jane is a 37-year-old woman who has presented to your clinic for the first time. She has a facial rash shown in the following photograph. According to her, the rash is more prominent when she is exposed to sunlight. She also complains of 4-kg weight loss in the past 3 months, occasional low-grade fever, and fatigue. She has had intermittent painful and mildly swollen wrists and knees to which recent finger pain has been added. On examination, oral ulcers that she has never noticed before are found. You order laboratory tests, which are remarkable for mild normocytic, normochromic anemia; mild thrombocytopenia; an elevated ESR of 110; positive ANA, and positive ds-DNA antibody. Which one of the following is the most appropriate long-term treatment option for her?

A. Naproxen.
B. Systemic corticosteroids.
C. Hydroxychloroquine.
D. Methotrexate.
E. Cyclophosphamide.

A

Correct Answer Is C.

The characteristic malar rash seen in the photograph that is sensitive to sun exposure, and other clinical and laboratory findings in this woman are suggestive of systemic lupus erythematosus (SLE). SLE is a chronic multisystem autoimmune disease 9 times more common in women than men. The disease has a relapsing-remitting nature and patients
experience episodes of exacerbations interspersed with periods of absent or relatively mild symptoms.

Clinical presentation of SLE is diverse because the disease has the potential to involve any organ system virtually. Patients often present with problems of the skin, mucosa, joints, CNS, kidneys, and hematological aberrancies.
According to current criteria, diagnosis of definitive SLE requires 4 or more of the following 11 criteria with at least one clinical and one laboratory, with the exception of biopsyproven LN (which requires fewer criteria).
Criteria are cumulative and need not be present concurrently.
1. Acute of subacute cutaneous lupus:
2. Chronic cutaneous lupus
3. Oral or nasal ulcers
4. Non-scarring alopecia
5. Synovitis involving 2 or more joints
6. Serositis (e.g. pericarditis, pleuritis)
7. Renal involvement i.e., lupus nephritis
8. Neurologic manifestations
9. Hemolytic anemia
10. Leukopenia (<4000/mm3) or lymphopenia (<1000/mm3)
11. Thrombocytopenia (<100,000/mm3)

There are also immunologic criteria associated with specific presentations of SLE including:
**Positive ANA: acute of subacute lupus
**Positive Anti-ds-DNA antibody: chronic cutaneous lupus
**Positive Anti-Sm antibody: oral and/or nasal ulcers
**Low complement (C3, C4.CH50): non-scarring alopecia
**Positive direct Coombs’ test: synovitis
**Anti-phospholipid antibody: serositis

This woman has 3 clinical criteria (malar rash, mouth ulcers, and joint pain), one clinical criterion (thrombocytopenia), and 2 immunological criteria (positive ANA and ant- ds-DNA antibody). She has a definitive diagnosis of SLE.

Management of SLE often depends on disease severity and disease manifestations; however, hydroxychloroquine is the cornerstone of treatment in all SLE patients and should be
used as the most appropriate long-term treatment option for Jane. Hydroxychloroquine is associated with decreased flare-ups and survival benefits in SLE patients. It is welltolerated and has few adverse effects. One concern is the risk of ocular toxicity that can be eliminated with appropriate dosing.
Nonsteroidal anti-inflammatory drugs (NSAIDs) such as naproxen (option A) or low-dose systemic steroids can be considered for SLE patients with mild cutaneous lupus,
tenosynovitis, or serositis. In long term, however, such drugs are associated with undesirable adverse effects.

A number of steroid-sparing immunosuppressive drugs beyond hydroxychloroquine are used to treat severe lupus manifestations other than skin manifestations, tenosynovitis, and
serositis, the involvement of which is considered mild lupus.

For example, cyclophosphamide (option E) and mycophenolate are medications of choice for lupus nephritis. Janet does not have renal involvement and such drugs are not indicated for her.

Methotrexate (option D) is another steroid-sparing agent that can be used in SLE patients.

47
Q

Mary is 32 years old and 36 weeks pregnant. She has had an uneventful pregnancy to date and has attended her antenatal care regularly. At today’s visit, fetal auscultation is significant for bradycardia. A CTG confirms a persistently decreased fetal heart rate of 67 bpm. Due to the risk of fetal jeopardy, emergency cesarean section is considered and performed. The neonate has still bradycardia at the rate of 70 bpm. Which one of the following antibodies is most likely to have contributed to such a presentation?

A. Anti-Ro antibody.
B. Anti-La antibody.
C. Anti-cardiolipin antibody.
D. Anti-Sm (Smith) antibody.
E. U1-ribonucleoprotein.

A

Correct Answer Is A

With the options all being immunologic markers, the question requires you to know the congenital rheumatologic disease associated with neonatal bradycardia, which is congenital (neonatal) lupus erythematosus (NLE).

Neonatal lupus erythematosus (NLE) is thought to be caused by the transplacental passage of maternal autoantibodies; however, only 1-2% of infants with positive maternal autoantibodies develop NLE. The most common clinical manifestations of NLE are dermatologic, cardiac, and hepatic. Some infants may also have hematologic, central nervous system, or splenic abnormalities.

Cardiac involvement in NLE is common, occurring in nearly 65% of patients. Rhythm abnormalities and conduction defects may be observed in various forms, but congenital complete heart block is the most common rhythm disturbance observed, with an incidence of 15-30%. Complete heart block can present with profound bradycardia resulting in
congestive heart failure. Cardiac blocks usually develop in utero between the 18th and 20th weeks of pregnancy. The risk is more significant for neonates born to mothers with primary Sjögren syndrome or undifferentiated autoimmune syndrome than those with systemic LE (SLE).

The mother produces immunoglobulin G autoantibodies against Ro (SSA), La (SSB), and/or U1-ribonucleoprotein (U1-RNP), and they are passively transported across the placenta.

The presence of maternal anti-SSA/Ro and anti-SSB/La antibodies increases the risk of bearing infants with NLE; rarely, NLE is due to maternal passage of U1-RNP antibodies.

These autoantibodies can be found alone or in combination; however, anti-Ro is present in almost 95% of patients and is most likely to have caused this problem.

Anti-La antibody (option B) and U1-ribonucleoprotein (option E) can also cause NLE but anti-Ro antibody is most likely to be detected.

There is no strong link between NLE and maternal anti-cardiolipin (option C) or anti-Sm (option D) antibody.

48
Q

A 60-year-old man presents to the Emergency Department with a complaint of a red and severely painful right big toe as shown in the following photograph. He has had such
episodes in the same toe and the left big toe before. Based on the history, physical examinations, and laboratory tests, he is diagnosed with an acute attack of gouty arthritis. He is
obese and has a BMI of 32. He drinks 5 to 6 standard units of alcohol every day. He also has hypertension currently controlled on hydrochlorothiazide. His lab results
include hypercholesterolemia, high LDL, low HDL, a serum creatinine level of 160 mmol/L (normal 50-110 mmol/L), and a serum uric acid level of 670 mmol/L (normal: 120-420
mmol/L) are remarkable. Which one of the following would you consider the first line for his treatment?

A. Prednisolone.
B. Codeine.
C. Naproxen.
D. Allopurinol.
E. Colchicine.

A

Correct Answer Is A

First-line medications for the treatment of acute gouty arthritis are NSAIDs and colchicine. Of NSAIDs, indomethacin and naproxen are the potent ones mostly prescribed for the
condition, but other NSAIDs such as ibuprofen or diclofenac can be effective too. The COX2 inhibitor celecoxib has approximately the same efficacy and can replace NSAIDs where
gastrointestinal bleeding makes use of traditional NSAIDs a concern.

Although naproxen (option C) is one of the first-line options for the treatment of acute gout, it should be avoided for this patient, as he has an increased serum creatinine level of 180
mmol/L indicating renal impairment.

Colchicine is another first-line medication for the treatment of acute gouty arthritis. Treatment starts with 1.2 mg statim, followed by 0.6 mg six hours later, and then 0.6 mg once or
twice daily for two to three days for those with normal renal function. In patients with reduced kidney function e.g. chronic kidney disease (CKD), who have a GFR of less than 50
ml/min/1.73m2, or those receiving CYP3A4 or P-glycoprotein inhibitors, the dose of colchicine should be reduced. Colchicine is better avoided or used with extreme caution (e.g.longer dose intervals) in patients with GFR<30 ml/ml/1.73m2.

Oral steroids, such as prednisolone or intra-articular steroids are also effective for the treatment of acute gouty arthritis but are reserved for patients who cannot tolerate or have contraindications to first-line medications. The recommended dose is 0.5mg/kg of prednisolone (or equivalent) for 5 to 10 days with gradual dose reduction. Intra-articular steroid injection is the preferred method with only one affected joint and contraindications to first-line agents.

For this patient, with renal impairment, either colchicine (option E) at a reduced dose or prednisolone can be used; however, since GFR is not known (not at least in the question),
prednisolone would be safer than colchicine and the preferred option here. It should be noted that (for the purpose of the exam) colchicine can be an appropriate option in the
absence of prednisolone.

(Option B) Codeine can relieve the pain but is not effective in counteracting the inflammation; hence, not an appropriate option.

(Option D) Allopurinol is a uric acid lowering agent and is indicated for patients with high serum uric acid levels as a measure of prophylaxis and prevention of further recurrences. Despite being effective in the prevention of further attacks, it has no role in managing acute gout.

49
Q

Sue, 56 years old, presents to the Emergency Department with acute-onset right knee pain starting 3 hours ago and worsening gradually. On examination, she has a BMI of 32, blood
pressure of 158/105 mmHg, pulse rate of 90 bpm, and temperature of 37°C. A joint aspirate excludes septic arthritis; however, the presence of birefringent crystals is highly suggestive of gouty arthritis. Her medical history is remarkable for chronic kidney disease due to long-standing hypertension. Laboratory tests are ordered, of which a serum uric
acid of 0.6 mmol/L (normal: 0.12-0.42 mmol/L) and a GFR of 30 (normal: 90-120 ml/min/1.73m2) are remarkable. Which one of the following is the most appropriate treatment to
consider for her?

A. Paracetamol.
B. Naproxen.
C. Codeine.
D. Colchicine.
E. Allopurinol.

A

Correct Answer Is D

This patient has an acute attack of gouty arthritis. The aims of treatment for gouty arthritis are the treatment of acute attacks and the prevention of further episodes.
Non-steroidal anti-inflammatory drugs (NSAIDs) or colchicine are first-line management options for first-time or recurrent acute gout attacks. Of the NSAIDs, indomethacin has been
traditionally used, as a potent NSAID, for the treatment of acute gouty arthritis. Naproxen is another potent NSAID for this purpose. These two are the most often used NSAIDs for
those under 60 years of age, for whom there is no contraindication to use of this drug class. Other less potent NSAIDs such as ibuprofen or diclofenac can be used as well. Some of
the contraindications to using NSAIDs are:

**History of peptic ulcer disease or gastrointestinal bleeding
**Chronic kidney disease
**Hepatic disease
**Any kind of other adverse drug reaction to NSAIDs

NOTE - the cyclooxygenase (COX) 2 inhibitor celecoxib has been used with success in the treatment of acute gouty arthritis with almost the same efficacy as traditional NSAIDs.

Celecoxib is the preferred option when NSAIDs are considered for treatment, but the patient has a history of GI bleeding, peptic ulcer disease, or gastritis.

Colchicine is another first-line medication for the treatment of acute gouty arthritis. Treatment starts with 1.2 mg statim, followed by 0.6 mg six hours later, and then 0.6 mg once or
twice daily for two to three days for those with normal renal function. This dosing is not associated with gastrointestinal adverse effects such as diarrhea and vomiting which were
commonly seen with higher doses and shorter intervals that were traditionally used.

NOTE - the 0.6 mg tablet size is not available in Australia and the 0.5 mg tablet is usually substituted.

For patients with reduced kidney function e.g., chronic kidney disease (CKD), who have a GFR of less than 50 ml/min/1.73m2, or those receiving CYP3A4 or P-glycoprotein inhibitors,
the dose of colchicine should be reduced. Colchicine is better avoided or used with extreme caution (e.g., longer dose intervals) in patients with GFR<30 ml/min/1.73m2.
Oral steroids are reserved for patients who cannot tolerate or have contraindications to first-line medications. The recommended is 0.5mg/kg of prednisolone (or equivalent) for 5 to 10 days with gradual dose reduction. Intra-articular steroid injection is the preferred method with only one affected joint and contraindications to first-line agents.
Sue has a GFR of 30 ml/min/1.73m2. For her, NSAIDs such as naproxen (option B) are contraindicated. For her, either reduced doses of colchicine or steroids (not an option) would
be the regimen of choice.

NOTE – if an option, oral or intra-articular steroid will be a better choice for acute gouty arthritis in patients with renal impairment.

Although paracetamol (option A) or codeine (option C) can provide pain relief, they do not have an anti-inflammatory effect, which is required for the treatment of gouty arthritis.

Allopurinol (option E) is a urate-lowering agent and is used for gout prophylaxis where indicated. It is not effective in the treatment of acute gout.

50
Q

Laura, 56 years old, presents to the Emergency Department with an acutely painful left index finger shown in the following photograph. On examination, she is afebrile, and the right
finger’s distal interphalangeal (DIP) joint is red, warm, and extremely tender to touch with decreased motion due to pain. She has had 4 episodes of such incidents in the past 2 years
in the same finger diagnosed as recurrent gouty arthritis. She is currently on a prophylactic dose of colchicine and allopurinol 100 mg daily. She is also a known case of longstanding hypertension, controlled on hydrochlorothiazide and metoprolol, and also a known case of hypertensive nephropathy. Which one of the following would you consider for her as the first-line treatment option at this stage?

A. Cessation of allopurinol.
B. Indomethacin.
C. Celecoxib.
D. Prednisolone for 10 days.
E. Increasing the dose of allopurinol to 150 mg, daily.

A

Correct Answer Is D

First-line treatment for patients with acute gouty arthritis is either potent NSAIDs such as indomethacin and naproxen or colchicine. The COX 2 inhibitor celecoxib is an acceptable
but off-lable option when NSAIDs are considered first line, but the patient has a history of peptic ulcer disease, gastrointestinal bleeding ,or gastritis.

NOTE - Since celecoxib is not approved for treatment of gout, it is better not to choose it as an option in the exam.

In general, oral steroids, such a prednisolone or intra-articular steroids should be considered for patients who cannot tolerate or have contraindications to first-line medications. The
recommended dose is 0.5mg/kg of prednisolone (or equivalent) for 5 to 10 days with gradual dose reduction. Intra-articular steroid injection is the preferred method with only one affected joint and contraindications to first-line agents. Colchicine in reduced dose (not an option here) can be used as well.
For this patient with renal impairment, prednisolone for 10 days is the most appropriate option.

(Option A) Allopurinol is a urate-lowering agent and used for prevention of further attacks of gouty arthritis. It should never be ceased during a gouty arthritis flare-up, as the
resultant fluctuating serum urate level may worsen the condition.

(Options B and C) NSAIDs such as indomethacin and the COX2 inhibitor celecoxib are contraindicated in patients with chronic kidney disease and should be avoided.

(Option E) Increasing the dose of allopurinol may considered later for prevention of further attacks but is not useful for treatment of acute flares. It is also recommended to avoid
changing the dose of allopurinol in patient who is already on it and has presented with an acute attack.

51
Q

A 58-year-old man presents to your GP clinic with an acutely painful left knee complaint. On examination, the left knee is warm, swollen, and tender. and erythematous. He is afebrile,
and the rest of the physical examination is inconclusive. A joint aspirate shows typical features of gouty arthritis. Which of the following is the most appropriate next step in the management of this patient?

A. Celecoxib.
B. Aspirin.
C. Colchicine.
D. Prednisolone.
E. Allopurinol.

A

Correct Answer Is C

For treatment of gouty arthritis, either non-steroidal anti-inflammatory drugs (NSAIDs) or colchicine are used as first-line options. Of NSAIDs, indomethacin and naproxen are most
potent and usually selected over other NSAIDs for acute treatment of gouty arthritis.
Of the options, colchicine can be used for this patient as the first-line medication (NSAIDs would be a better choice in options). Treatment starts with 1.2 mg statim, followed by 0.6
mg six hours later and then 0.6 mg once or twice daily for two to three days for those with normal renal function. In patients with reduced kidney function e.g. chronic kidney disease
(CKD), who have a GFR of less than 50 ml/min/1.73m2, or those receiving CYP3A4 or P-glycoprotein inhibitors, the dose of colchicine should be reduced. Colchicine is better
avoided or used with extreme caution (e.g. longer dose intervals) in patients with GFR <30 ml/ml/1.73m2.

(Option A) Celecoxib, a COX2 inhibitor, is categorized under NSAIDs. Although celecoxib is sometimes used off-label for patients with gout in whom traditional NSAIDs are
contraindicated due to gastrointestinal problems such as history of gastrointestinal bleeding, peptic ulcer disease or gastritis, it has not been officially approved for treatment of
gout. It should be noted that celecoxib still has the hepatic, renal, and cardiovascular toxic effects similar to NSAIDs.

(Option B) Although aspirin has anti-inflammatory effects, it can impair excretion of urate from kidneys and should be avoided in patients with gout.

(Option D) Oral steroids, such a prednisolone or intra-articular steroids should be considered for patients who cannot tolerate or have contraindications to first-line medications such
as NSAIDs and colchicine.

(Option E) Allopurinol is a uric acid lowering agent, and is indicated for patients with high serum uric acid levels as a measure of prophylaxis and prevention of further recurrences.
Despite being effective in prevention of further attacks, it has no role in management of acute gout.

52
Q

A 49-year-old woman presents to your GP clinic with progressive low back pain that is shooting and radiates to her left buttock. She denies any preceding trauma or sprain. Her past
medical history is significant for breast cancer treated with mastectomy, chemotherapy, and radiotherapy six year ago and osteoarthritis of the lumbosacral spine. On examination,
there is tenderness over the L4/L5 area, a positive straight leg raise (SLR) test on the left side. Which one of the following features in the clinical assessment is an indication for
further assessments of this patient?

A. Absence of a preceding trauma or sprain.
B. Osteoarthritis of the lumbosacral spine.
C. Positive SLR test.
D. The radicular nature of her pain.
E. Past history of breast cancer.

A

Correct Answer Is E

This patient has progressive back pain, shooting in nature, that makes radiculopathy a possibility.

Acute episodes of low back pain are very common and often managed conservatively if a thorough history and physical examination is negative for alarming signs, symptoms, and
risk factors. These alarming features are termed ‘red flags’ for low back pain.

In this patient, the history of breast cancer is an alarming feature and requires prompt further assessment because bone is one of the most common sites of metastasis. A history
of cancer, except non-melanoma skin cancers, is the strongest alarm in the history of any patient with back pain. Breast, prostate, lung, thyroid, and kidney cancers account for 80%
of metastases. Almost 60% of patients with multiple myeloma have skeletal lytic lesions present at diagnosis.

Pain is the most common symptom in of metastatic bone disease. In patients with history of cancer, sudden severe pain raises concern for pathologic fracture. Neurologic
symptoms from nerve root/ spinal cord compression or spinal instability could be another manifestation.

Where a pathologic fracture is suspected, plain radiography is often the initial imaging study. Detailed and more specific investigations are then considered based on the probability
diagnosis.

(Option A) Absence of a preceding trauma or sprain is not an indication for further evaluation because many episodes of non-specific mechanical back pain are not preceded by
trauma or sprain remembered by the patient.

(Options B) Osteoarthritis of the lumbosacral spine is not a red flags for back pain to mandate further assessment.

(Option C and D) Radicular nature of the pain, confirmed by the SLR test, is not an indication for further assessment of this patient. In general, radicular pain is managed
conservatively with analgesia, physiotherapy and maintained daily physical activity. For radicular pain without alarming features, MRI is not necessary unless the presentation is
unusual, such as sign and symptoms that cannot be attributed to only one nerve root involvement.

TOPIC REVIEW
Red flags for low back pain are summarized in the following table:

53
Q

A 68-year-old patient of yours presents for medical evaluation. He is a known-case of benign prostatic enlargement. Today he is complaining of urinary problems in form of difficulty starting micturition and dribbling at the end of urination, as well as constipation. He also has back pain and left thigh pain. On examination, there is decreased sensation over the
medial aspect of the left lower leg. Which one of the following indicates intervertebral disc prolapse in this patient?

A. Urinary problems.
B. Thigh pain.
C. Back pain.
D. Decreased sensation of the lower leg.
E. Constipation.

A

Correct Answer Is D

Of the options, only decreased sensation of the lower leg can be specific to an intervertebral disc prolapse and others can be manifestations of non-neurological problems as well.
Radicular lower back pain can happen due to chemical or mechanical irritation of nerve roots. The pain is sharp, shooting and has an electric quality. Unilateral leg pain is the more
pronounced symptoms and is often worse than pain in the back. Pain concentrates distally, running into the lower limb, usually extending below the knees. Pain, numbness and
paresthesia follow a dermatomal distribution. Reflexes may be reduced or even absent. Motor weakness not always would be present.
Lesions of the cauda equina can present with back pain, leg pain, paresthesia around the anus, and urinary incontinence. However, in this patient, the urinary problem is described as
difficulty starting micturition and terminal dribbling, which is consistent with bladder outlet obstruction by causes such as an enlarged prostate. Urinary problems (option A) of this patient is very unlikely to have been caused by an intervertebral disc prolapse.

Although disc prolapse in the lumbar area can present with back pain (option C) and thigh pain (option B), such pain may have been caused by other factors than radiculopathy.

Back pain may be due to a mechanical stressor. In fact, back pain is the common factor between mechanical and neural pain in the back. The accompanying symptoms are the only
indicators helping to differentiate between the two. Leg pain may have other causes such as mechanical injuries and vascular problems (e.g., venous insufficiency, arterial disease,
etc.). Neural impairments other than radiculopathy may give rise to pain as well.

Constipation (option E) in this patient may have many other causes as well, with one being bladder obstruction and pressing of a distended bladder on the rectum. Decreased
dietary fibers, limited physical activity, and many other factors could have caused the constipation. Disc prolapse is more likley to be associated with fecal incontinence rather than
constipation.

54
Q

Which one of the following statements is correct regarding osteoarthritis (OA)?

A. Immobilization of the affected joint at the early stages of the disease slows further damage.
B. OA is the disease of the subchondral bone that progresses to involve the overlying cartilage later in the course of the disease.
C. OA is often associated with morning stiffness of over 30 minutes.
D. Secondary OA commonly affects the metacarpophalangeal joints.
E. OA can develop secondary to rheumatoid arthritis (RA)

A

Correct Answer Is E

OA characteristically starts with thinning and destruction of the cartilage (not the bone). Less protection due to loss of cartilage renders the underlying bones susceptible to
repetitive traumas resulting in bone destruction and impaired bone healing and remodeling.

OA has two types:
Idiopathic (primary) OA - idiopathic OA comprises the majority of cases. Joint involvement is usually symmetrical. The most common joint involved is the first carpometacarpal joint of the thumb (base of the thumb), followed by the first metatarsophalangeal of the big toe and the distal interphalangeal (DIP) joints of the hand.
Other joints that may be affected
are the proximal interphalangeal joints of the hand, the knees, the hips, acromioclavicular joints, and spinal joints especially the facet joints between C5 and C6, C6 and C7, L3 and
L4, and L4 and L5.

Generally, primary OA is more common in women (with the exception of OA of the hip, which affects both sexes equally)

Secondary OA - in secondary OA, there is an underlying cause for OA. Joints are usually affected unilaterally, with the knee being the most commonly affected joint. Secondary OA is
more common in men. Secondary OA typically involves joints with pres-existing damage such as those with destructions and deformities caused by rheumatoid arthritis, chronic
gout, etc. Generally speaking, any structural abnormality makes a joint prone to OA.

The clinical presentation of OA is pain that comes on with activity and is relieved by rest. The pain often becomes worse throughout the day. If there is morning stiffness, it should
be less than 30 minutes.

The first-line pharmacological treatment for OA is simple analgesics such as paracetamol. It is strongly recommended that paracetamol be taken on a regular basis, even when
there is no pain, rather than on an as-needed basis because the former provides more pain control.

NSAIDs are used as second-line medications only if:
**Optimal therapy with paracetamol fails to control the pain
**There is evidence of inflammation, such as pain that is worse with rest, nocturnal pain, and gelling of the joint (stiffness with rest)

Immobilization of the affected joint is never a part of the management plan because It makes the affected joint stiffer and the condition worse.

55
Q

A 47-year-old woman presents to your clinic with an 8-month right ankle swelling and pain history. The patient relates that the ankle is stiff in the morning and takes over one hour of
walking and physical activity to ease up. No other joint is involved. On examination, she has a temperature of 36.9°C with other vital signs within normal limits. The right ankle joint is
swollen and has a restricted range of motion and is slightly tender. An X-ray of the right ankle and foot shows periarticular osteopenia and narrowed joint space of the right ankle.
Which of the following could be the most likely underlying cause of this presentation?

A. Septic arthritis.
B. Osteomyelitis.
C. Systemic lupus erythematosus (SLE).
D. Rheumatoid arthritis (RA).
E. Osteoarthritis (OA).

A

Correct Answer Is D

This patient has a painful and swollen right ankle joint associated with a morning stiffness of over one hour which indicates there is an inflammatory process either infective or
rheumatologic involved. Osteoarthritis (OA) (option E) is not inflammatory and unlike inflammatory arthritides is aggravated by activity and relieved by rest. Joint space narrowing can be a feature of OA, but subchondral sclerosis and cysts rather than periarticular osteopenia/osteoporosis are expected on x-ray.

X-ray changes in OA of weight-bearing joints such as the hip, knee, and ankle include:
**Joint space narrowing
**Osteophytes
**Subchondral sclerosis and cysts

Septic arthritis (option A) has an acute presentation with fever, joint pain, and tenderness, swelling, warmth, and redness in a matter of hours to days. An afebrile patient with joint pain and swelling of 8 months duration is very unlikely to have septic arthritis.
X-ray findings in septic arthritis include the following:
**X-rays may be normal in the very early stage of the disease
**The joint effusion may be seen
**Juxta-articular osteoporosis due to hyperemia
**Joint space narrowing due to cartilage destruction in the acute phase
**Destruction of the subchondral bone on both sides of a joint
**If left untreated, reactive juxta-articular sclerosis and, in severe cases, ankylosing will develop

Local presentations of osteomyelitis (option B) are usually seen in the affected bone rather than the adjacent joint. Osteomyelitis has an insidious onset with constitutional
symptoms such as fatigue, fever, chills, local bone tenderness, etc. Early plain radiological findings in osteomyelitis are subtle and not often obvious until 5-7 days from the onset in
children and 10-14 days in adults. Radiographs taken after this time may show the following:
**Effusion at the adjacent joint (usually the only joint finding)
**Regional osteopenia of the affected bone
**Periosteal reaction (often thickening or elevation caused by periostitis)
**Focal bony lysis or cortical loss
**Loss of trabecular architecture
**New bone formation
**Eventual peripheral sclerosis.

This patient’s, history, physical examination, and radiological findings are inconsistent with osteomyelitis as a diagnosis.

SLE (option C) is a multisystem rheumatologic disease with joints involved in a large number of patients (70-90%). Often two or more joints are involved. SLE arthritis is non-erosive,
meaning that no erosive lesions or joint space narrowing or destruction is expected on the x-ray of an SLE-affected joint. This patient has joint space narrowing which is inconsistent
with SLE. Expected X-ray findings of an affected joint in SLE include:
**Soft-tissue swelling
**Periarticular (juxta-articular) osteopenia or osteoporosis.
**Preserved joint space

Of the options, only RA best fits the x-ray findings described in the scenario. X-ray findings of an affected joint in RA include the following:
**Soft tissue swelling (mostly synovium)
**Joint space narrowing or even destruction in more advanced cases
**Erosions of joint margins (in more advanced cases)
**Periarticular (juxta-articular) osteopenia or osteoporosis.

**NOTE – Typically, patients with RA arthritis present with symmetrical arthritis affecting the wrists and the metacarpophalangeal and proximal interphalangeal joints of the
hands. Involvement of the metatarsophalangeal joints of the feet **

56
Q

Which one of the following is NOT an expected physical finding in infectious tenosynovitis of finger?

A. Pain on passive flexion of the affected finger.
B. Pain on passive extension of the affected finger.
C. Finger in fixed flexion.
D. Fusiform swelling of the whole finger.
E. Tenderness along the flexor tendon sheath.

A

Correct Answer Is A

Flexor sheath infections, also known as infective tenosynovitis, are relatively common hand infections.

Digital flexor sheaths are a closed continuous synovial system that contain the flexor digitorum profundus and flexor digitorum superficialis. Flexor sheath infections are caused by direct inoculation due to trauma, spread of infection from adjacent tissues, or via hematogenous spread. Common injuries include animal bites, puncture wounds, IV drug use, and wounds exposed to fresh/saltwater. Such injuries, if associated with severe tenderness, should raise a high suspicion of flexor tenosynovitis.

Flexor sheath infections begin with buildup of exudative fluid within the tendon sheath, followed by the development of purulent fluid.
Bacterial overgrowth within the synovial sheath
leads to increased fluid and pressure, leading to ischemia, and resulting in a septic necrosis of the tendon sheath and surrounding structures. If left untreated, complications may
include soft tissue necrosis, osteomyelitis, and necrotizing fasciitis.

Infective flexor tenosynovitis presents with the four cardinal signs referred to as Kanavel signs:
**Flexed resting position of the affected finger
**Fusiform swelling of the whole finger
**Tenderness along the flexor tendon sheath
**Pain (usually exquisite) with passive extension of the affected finger

Of the options, pain on passive flexion of the affected finger is not a feature.

57
Q

Which one of the following is the earliest sign of flexor sheath infection?

A. Fusiform swelling of the finger.
B. Pain on passive extension of the affected finger.
C. Fever.
D. Tenderness along the tendon sheath.
E. Flexed resting position of the affected finger

A

Correct Answer Is B

Flexor sheath infections, also known as infective tenosynovitis, is a serious hand infection which if not attended to urgently can result in a septic necrosis of the tendon sheath and
surrounding structures. If left untreated, complications may include soft tissue necrosis, osteomyelitis, and necrotizing fasciitis.
Flexor sheath infections are caused by direct inoculation due to trauma, spread of infection from adjacent tissues, or via hematogenous spread.

Common signs of flexor sheath infections are:
**Flexed resting position of the affected finger
**Fusiform swelling of the whole finger
**Tenderness along the flexor tendon sheath
**Pain (usually exquisite) with passive extension of the affected finger

These signs are also known as Kanavel signs. Collectively, Kanavel signs have a specificity of over 90% for diagnosis of flexor sheath infection.

Of these signs, the earliest is usually pain on passive infection of the affected sign. A later sign is tenderness along the flexor sheath (option D) that suggest proximal spread of the
infection, followed by fusiform swelling (option A) and fixed resting position of the finger (option E). Fever (option C) is often absent but if present indicates systemic spread of the
infection which happens late in the course of the disease if left untreated.

58
Q

Ruth, 70 years of age, has presented to your office with a complaint of muscle weakness for the past couple of weeks. She has hypertension and coronary artery disease for which
she is on ramipril and atorvastatin. Physical examination is remarkable for bilateral shoulder girdle muscle weakness and tenderness. You order blood tests that are significant for an elevated ESR of 48 mm and normal creatine kinase (CK). Which one of the following is the most likely diagnosis?

A. Polymyositis.
B. Statin-induced myositis.
C. Dermatomyositis.
D. Polymyalgia rheumatica.
E. Adhesive capsulitis.

A

Correct Answer Is D

Polymyositis (option A), statin-induced myositis (option B), and dermatomyositis (option C) all can cause inflammation and damage of the muscles, primarily proximal muscles,
with elevated inflammatory markers such as ESR and CRP as well CK as a result. For Ruth, a normal CK excludes such conditions with rather a high certainty.

Adhesive capsulitis (option E), if bilateral, can present with rather similar physical findings but inflammatory ESR and CRP are normal.
Polymyalgia rheumatica (PMR) on the other hand fits the scenario. PMR is a relatively common chronic inflammatory condition of unknown etiology that affects elderly individuals.

It is characterized by symmetrical myalgia of the hip and/or shoulder girdles with accompanying morning stiffness lasting more than 45 minutes. PRM is a diagnosis of exclusion.

Laboratory findings in PMR include elevated ESR (often above 40) and CRP, and normocytic normochromic anemia.

59
Q

The 9-year-old Tim is brought to the Emergency Department by his mum after he fell off his bike and hurt his left shoulder. The left shoulder is painful, and he is barely moving his
left arm due to the pain. On physical examination, there is no bruise or deformity of the left shoulder, but there is tenderness over the middle third of the left clavicle. Neurovascular examination of the left arm and hand is normal. Which one of the following is the most appropriate next step investigation?

A. X-ray of the left clavicle; PA view.
B. X-ray of the left clavicle; AP view.
C. X-ray of both clavicles; AP view.
D. CT scan.
E. X-ray of the left clavicle; oblique view.

A

Correct Answer Is B

Tim has clinical features of fracture/injury of the middle third of his left clavicle. Clavicular fractures are common in children, often a as result of a fall on outstretched arm. When
clinical suspicion of clavicle exists, an standard anteroposterior (AP) x-ray and AP with 15° cephalic tilt is the initial and, in most cases, the only investigation needed.

The AP film visualizes the entire clavicle including its articulation with the acromion and the sternum. AP with cephalic tile straightens out the clavicle and is used to better visualize
occult fractures.

CT scan (option D) may be considered in fractures of the medial third of the clavicle for assessment of potential tracheal impingement and thoracic anatomy, or in displaced lateral
third injuries to assess the coracoclavicular ligament.

PA views (option A) and obliques views (option E) add no diagnostic value and are not recommended for suspected clavicular fractures. AP views of both clavicles (option C) are indicated only of comparison is required or the injury to both clavicles is suspected.

60
Q

A 52-year-old man comes to your GP practice complaining of difficulty holding and grasping objects. He has smoked 20 sticks per day for the past 15 years but is otherwise well.
You immediately notice his hands as shown in the photo below. On further probing, the patient tells you that he has similar findings on his feet. Which of the following is the most
likely diagnosis?

A. Buerger Disease.
B. Wegener’s polyangiitis.
C. Raynaud phenomenon.
D. Diabetes mellitus.
E. Cogan’s Syndrome.

A

Correct Answer Is A

The clinical picture is suggestive of thromboangiitis obliterans, also known as Burger’s disease, as the most likely diagnosis.
Thromboangiitis obliterans also called Buerger disease is a nonatherosclerotic, segmental, inflammatory disease that most commonly affects the small to medium-sized arteries
and veins of the extremities, resulting in occlusive thrombus and distal extremity ischemia It occurs most commonly in men, with the age of onset in the 40s, is strongly associated with smoking, and is often progressive. Amputation is commonly required to manage pain or secondary infection.

Wegener’s polyangiitis or granulomatosis (option B) with polyangiitis is a rare vasculitis of unknown cause. The classic triad is (1) upper respiratory tract granuloma, (2) fleeting
pulmonary shadows (nodules), and (3) glomerulonephritis, none of which is present in this scenario.

Raynaud phenomenon (option C) is a clinical syndrome of episodic arteriolar vasospasm usually involving the fingers and toes (one or two at a time). The characteristic finding in
the Raynaud phenomenon is being triggered by cold temperatures, anxiety, or stressful events. The fingers and toes change from blue to red to a very pale appearance. It may also
involve the nose, ear, or nipple. It does not cause or result in digital amputation.
Diabetes mellitus is an established and common underlying cause of digital amputation. However, it is unlikely in this scenario in the absence of a history or suggestive clinical
findings (option D).

Cogan’s syndrome (option E) is a rare autoimmune vasculitis that affects the eyes and inner ears. Symptoms of the syndrome include irritation and pain in the eyes, decreased
vision, hearing loss, and vertigo.

61
Q

A 30-year-old female with severe muscle aches and fatigue for the past 4 months. She denies fever or weight loss but admits to mood and sleep disturbances. On examination,
there are 13 tender points in different areas including the neck, upper chest, inner aspects of the knee, upper thighs, and lateral epicondyles. ESR, CRP, and CK are normal. Which of the following could be the most appropriate pharmacological treatment option?

A. Paracetamol.
B. Naproxen.
C. Amitriptyline.
D. Prednisolone.
E. Fluoxetine.

A

Correct Answer Is C

For this patient, fibromyalgia seems to be the best-fitting explanation for the clinical presentation. Fibromyalgia is characterized by widespread pain affecting all four body
quadrants, along with systemic symptoms like fatigue, sleep problems, and cognitive impairment. It is more common in females with a female-to-male ratio of 4:1. The usual age of
onset is between 29- 37 years, but it is often diagnosed by around 44 -53 years. There may be a family history.
Pain should be present for at least 3 months and there must be 11 out of 18 characteristic tender points for a diagnosis.

Fibromyalgia has multiple, varied, and fluctuating symptoms. It should be considered in a patient with chronic musculoskeletal pain, fatigue, and poor sleep. These symptoms are
usually accompanied by depression or anxiety, sensitivity to chemicals, irritable bowel, or restless legs. The symptoms often start or worsen during a period of severe psychosocial
or physical stress. Thorough examination and investigation are warranted to rule out other pathological causes of disease, especially in the presence of red flags which are as
follows:
**Older age at new symptom onset
**Weight loss
**Night pain
**Focal pain
**Fever or sweats
**Neurological features
**History of malignancy

Management includes explanation and support. Stress management and CBT are beneficial. Concomitant depression should be treated. Of the given pharmacological treatment
options, amitriptyline seems to be the most beneficial for short-term pain management.
Tricyclic anti-depressants (TCAs) such as amitriptyline and serotonin-noradrenaline reuptake inhibitors (SNRIs) such as duloxetine seem to be of short-term value for pain control in fibromyalgia. Low-dose TCAs or SNRIs elevate levels of serotonin and noradrenaline in the descending inhibitory nociceptive pathways of the CNS, providing a significant benefit in
fibromyalgia independent of effects on mood.
SSRIs such as fluoxetine (option E) can be used for the treatment of underlying or concomitant depression but are not as effective as TCAs and SNRIs for pain control and
symptomatic relief for fibromyalgia.

Paracetamol (option A) can be given initially but is not the most appropriate treatment. European League Against Rheumatism (EULAR) recommends the use of simple analgesics
like paracetamol; however, there is insufficient data to back this. Paracetamol use has not been studied in fibromyalgia patients, other than in combination with tramadol, where the
combination resulted in a modest (18%) improvement in pain.

NSAIDs such as naproxen (option B) are of no proven benefit, and steroids such as prednisolone (option D) are not indicated in the condition.

62
Q

A 56-year-old woman presents with swelling in front of her right ear. She also complains of dry itchy eyes and a dry mouth. On examination, right parotid and xerostomia are
remarkable findings. Fine needle aspiration cytology (FNAC) is performed which is significant for lymphocytes without any clues of malignancy. Which diagnostic test would be the
best diagnostic test to confirm the diagnosis?

A. Salivary gland biopsy.
B. Schirmer test.
C. CT scan of the neck.
D. Ultrasound scan of the swelling.
E. ANA.

A

Correct Answer Is A

Sjӧgren syndrome is a chronic inflammatory disease characterized by lymphocytic infiltration and fibrosis of exocrine glands, especially lacrimal and salivary glands. The disease is much more prevalent in women (female: male~ 9:1), with the usual age of onset being in the 4th and 5th decades of life.

  • Rheumatoid arthritis
  • Systemic lupus erythematosus
  • Scleroderma

Considering the overlap of Sjӧgren syndrome with many other rheumatic disorders, it is sometimes difficult to determine whether a clinical manifestation is solely a result of Sjӧgren
syndrome or is due to one of the overlapping disorders.

Clinical features of the disease include the following:
* Dry eyes (xerophthalmia) and keratoconjunctivitis sicca due to decreased tear production- the patient may complain of feeling as sand under the eyelid or itchy eyes.
* Dry mouth (xerostomia) due to decreased salivation – dry mouth can lead to the following manifestations:
-Red smooth and dry tongue
-Severe and progressive dental caries
-Cracks at the corners of the mouth
-Chronic oral candidiasis
* Parotid gland swelling

NOTE - Dry eyes and mouth are the most common presenting features in adults, whereas, children often present with parotid gland enlargement.
Other clinical features of Sjӧgren disease include:
* Nasal dryness – can result in discomfort and bleeding
* Vaginal dryness – can result in dyspareunia, vaginitis, and pruritus
* Myalgia and fatigue
* Arthralgia or arthritis – similar to that of SLE with symmetrical involvement of small joints. arthritis is of non-erosive nature
* Raynaud’s phenomenon
* Recurrent miscarriages or stillbirths in women and a history of venous or arterial thrombosis related to the presence of antiphospholipid antibodies (e.g. lupus
anticoagulant or anticardiolipin antibodies)
* Leukopenia, anemia
* Lymphadenopathy
* Non-Hodgkin lymphoma

Currently, a minor salivary gland biopsy is the best single test to establish a diagnosis of Sjögren syndrome. In this procedure, an incision is made on the inner lip, and some minor salivary glands are removed for examination. In patients with a possible diagnosis of this disease but with severe extraglandular symptoms, a lip biopsy is often performed to firmly establish the diagnosis of Sjögren syndrome.

While this is the most definitive test, performing it is not necessary from a clinical standpoint. Patients with Sjögren syndrome are essentially treated symptomatically and are
observed for the development of other rheumatic disorders or lymphoma. This can be initiated without performing a biopsy. If the diagnosis is in doubt, or a definitive diagnosis is
needed, a biopsy is the best test.

(Option B) Schirmer test shows decreased tear production. A strip of filter paper is put under the lower eyelid, and the distance along the paper that tears are absorbed is measured.
Less than 5mm in 5 minutes is considered positive. It is one of the tests routinely performed when Sjӧgren disease is suspected but it is not diagnostic.

(Options C and D) Imaging studies are not diagnostic; however, they may be sometimes needed, particularly with extraglandular manifestations of the disease.

(Option E) ANA, particularly anti-Ro (SSA) and anti-La (SSB) may be positive, but again these could be elevated in other conditions as well.

63
Q

A 66-year-old man presents to your practice with complaints of left-sided cheek swelling, joint pain, and fatigue for the past 2 months. He has a 20-year history of type II diabetes
mellitus. He also mentions eye and mouth dryness for the past 6 months. On examination, there is non-tender swelling of the left parotid gland, several dental carries, and reddened eyes. Which one of the following is the most likely diagnosis?

A. Left salivary duct stone.
B. Sjӧgren syndrome.
C. Parotid gland carcinoma.
D. Chronic parotitis.
E. Behcet’s syndrome.

A

Correct Answer Is B

The clinical findings of dry mouth (xerostomia), dry eye (xerophthalmia), fatigue, and joint pain are strongly consistent with Sjӧgren’s syndrome as the most likely diagnosis.

Sjӧgren syndrome is a chronic inflammatory disease characterized by lymphocytic infiltration and fibrosis of exocrine glands, especially lacrimal and salivary glands. The disease is much more prevalent in women (female: male~ 9:1), with the usual age of onset being in the 4th and 5th decades of life.

Sjӧgren syndrome for which no underlying etiology can be identified is termed primary, whereas when it is secondary to other connective tissue disorders, it is called secondary Sjogren syndrome.

Considering the overlap of Sjӧgren syndrome with many other rheumatic disorders, it is sometimes difficult to determine whether a clinical manifestation is solely a result of Sjӧgren
syndrome or is due to one of its overlapping disorders.

Extra-glandular manifestations of Sjögren syndrome are common and include:

Cutaneous manifestations
* About 50% of patients with Sjögren syndrome have cutaneous findings, such as dry skin (xeroderma), palpable and nonpalpable purpura, and/or urticaria. Blepharitis and
erythema annulare may be other features.

Gastrointestinal manifestations
* Difficulty in swallowing (dysphagia) due to dryness of the pharynx and esophagus - Patients usually describe food becoming stuck in the upper throat.
* Lack of saliva may lead to impaired acid clearance and may result in gastroesophageal reflux and esophagitis.
* Abdominal pain and diarrhea can occur
* Rarely, patients develop acute or chronic pancreatitis and malabsorption due to pancreatic insufficiency.
* Patients with Sjögren syndrome are at increased risk for delayed gastric emptying, which can cause early satiety, upper abdominal discomfort, nausea, and vomiting.

Pulmonary manifestations
* The dryness of tracheobronchial mucosa (xerotrachea) can manifest as a dry cough.
* Less often, there might be mild dyspnea from trivial interstitial lung disease.
* Recurrent bronchitis or even pneumonitis (infectious or noninfectious)
* * Cardiac manifestations**Pericarditis and pulmonary hypertension can occur in Sjögren syndrome.
* Orthostatic symptoms related to dysfunction of autonomic control of blood pressure and heart rate are associated with increased severity of Sjögren syndrome.

Neurologic Manifestations
* Different studies estimate the prevalence of CNS symptoms in those with Sjogren syndrome from 8 to 40%. When CNS manifestations are present, other possible causes including concomitant SLE, multiple sclerosis, cerebrovascular disease, and Alzheimer’s disease should be assessed and meticulously investigated.

Renal manifestations
* The most common form of renal involvement in Sjögren syndrome is interstitial nephritis. Renal calculi, renal tubular acidosis, osteomalacia, nephrogenic diabetes
insipidus, and hypokalemia can occur secondary to tubular damage caused by interstitial nephritis,
* Interstitial cystitis, with dysuria, frequency, urgency, and nocturia, is strongly associated with Sjögren syndrome.
* Glomerulonephritis can be caused by Sjögren syndrome but is uncommon and is usually attributable to another disorder, such as SLE or mixed cryoglobulinemia

Other possible manifestations
* Nasal dryness – can result in discomfort and bleeding
* Vaginal dryness – can result in dyspareunia, vaginitis, and pruritus
* Myalgia and fatigue
* Arthralgia or arthritis – similar to SLE with symmetrical involvement of small joints. arthritis is of non-erosive nature
* Raynaud’s phenomenon
* Recurrent miscarriages or stillbirths in women and a history of venous or arterial thrombosis related to the presence of antiphospholipid antibodies (e.g. lupus
anticoagulant or anticardiolipin antibodies)
* Leukopenia, anemia
* Lymphadenopathy
* Non-Hodgkin lymphoma

(Options A, C, and D) Left salivary duct stone can lead to swelling of the left parotid gland, as can chronic parotitis and parotid gland carcinoma, but dry mouth, dry eye, and
arthralgia would be absent in these conditions.

(Option E) Behcet’s disease is a systemic vasculitis that presents mostly with mouth and genital ulcers as well as the involvement of other organs. Xerophthalmia and xerostomia
are not the features seen in Behcet’s disease. Chronic parotitis has a different clinical picture.

64
Q

A 43-year-old woman complains of increasing sensation of foreign body in her eyes, eye itching, and dry mouth for the past 6 months. She also reveals that she has recently developed a dry cough and pain in the hand joints. On examination, there is mild bilateral swelling of the parotid glands. The metacarpophalangeal (MCP) and proximal interphalangeal (PIP) are tender to touch, but not warm or swollen. She denies any morning stiffness. Serologic tests are significant for a positive ANA and RF. ESR and CRP are elevated. Urinalysis is normal. Which one of the following would be the most likely diagnosis?

A. Systemic lupus erythematosus.
B. Sjӧgren syndrome.
C. Bulimia nervosa.
D. Mumps.
E. Uveoparotid fever.

A

Correct Answer Is B.

The presentation of dry eye (xerophthalmia), dry mouth (xerostomia), and parotid gland enlargement are pointers toward Sjӧgren syndrome as the most likely diagnosis.

Sjӧgren syndrome is a chronic inflammatory disease characterized by lymphocytic infiltration and fibrosis of exocrine glands, especially lacrimal and salivary glands. The disease is much more prevalent in women (female: male~ 9:1), with the usual age of onset being in the 4th and 5th decades of life.

Sjӧgren syndrome for which no underlying etiology can be identified is termed primary, whereas when it is secondary to other connective tissue disorders, it is called secondary Sjӧgren syndrome.

Considering the overlap of Sjӧgren syndrome with many other rheumatic disorders, it is sometimes difficult to determine whether a clinical manifestation is solely a result of Sjӧgren syndrome or is due to one of its overlapping disorders.

Extra-glandular manifestations of Sjӧgren syndrome are common and include:

Cutaneous manifestations
* About 50% of patients with Sjögren syndrome have cutaneous findings, such as dry skin (xeroderma), palpable and nonpalpable purpura, and/or urticaria. Blepharitis and
erythema annulare may be other features.

Gastrointestinal manifestations
* Difficulty in swallowing (dysphagia) due to dryness of the pharynx and esophagus - patients usually describe food sticking in the throat.
* Lack of saliva may lead to impaired clearance of acid and may result in gastroesophageal reflux and esophagitis.
* Abdominal pain and diarrhea can occur
* Rarely, patients develop acute or chronic pancreatitis and malabsorption due to pancreatic insufficiency
* Patients with Sjögren syndrome are at increased risk for delayed gastric emptying, which can cause early satiety, upper abdominal discomfort, nausea, and vomiting.

Pulmonary manifestations
* Dryness of tracheobronchial mucosa (xerotrachea) that can manifest as a dry cough
Less often, there might be mild dyspnea from a trivial interstitial lung disease
* Recurrent bronchitis or even pneumonitis (infectious or noninfectious)

Cardiac manifestations
* Pericarditis and pulmonary hypertension can occur in Sjögren syndrome
Orthostatic symptoms related to dysfunction of autonomic control of blood pressure and heart rate are associated with increased severity of Sjögren syndrome

Neurologic Manifestations
* Different studies estimate the prevalence of CNS symptoms in those with Sjӧgren syndrome from 8 to 40%. When CNS manifestations are present, other possible causes including concomitant SLE, multiple sclerosis, cerebrovascular disease, and Alzheimer’s disease should be assessed and meticulously investigated.

Renal manifestations
* The most common form of renal involvement in Sjögren syndrome is interstitial nephritis. Renal calculi, renal tubular acidosis, osteomalacia, nephrogenic diabetes
insipidus, and hypokalemia can occur secondary to tubular damage caused by interstitial nephritis,
* Interstitial cystitis, with symptoms of dysuria, frequency, urgency, and nocturia is strongly associated with Sjögren syndrome.
* Glomerulonephritis can be caused by Sjögren syndrome but is uncommon and is usually attributable to another disorder, such as SLE or mixed cryoglobulinemia.

Other possible manifestations
**Nasal dryness – can result in discomfort and bleeding
**Vaginal dryness – can result in dyspareunia, vaginitis, and pruritus
**Myalgia and fatigue
**Arthralgia or arthritis – similar to that of SLE with symmetrical involvement of small joints. arthritis is of non-erosive nature
**Raynaud’s phenomenon
**Recurrent miscarriages or stillbirths in women and a history of venous or arterial thrombosis related to the presence of antiphospholipid antibodies (e.g. lupus
anticoagulant or anticardiolipin antibodies)
**Leukopenia, anemia
**Lymphadenopathy
**Non-Hodgkin lymphoma

Laboratory findings may include (but are not limited to) elevated ESR, positive CRP, positive RF, and positive ANA (Anti Ro and Anti La).

(Option A) SLE can be a possible cause of Sjögren syndrome, especially with the presence of features such as positive ANA and symmetrical involvement of hand joints, but even so, the diagnosis would be Sjögren syndrome secondary to SLE because the dry mouth and dry eyes are not features commonly seen in SLE.

(Option C) Bulimia nervosa can cause bilateral parotid enlargement due to repetitive vomiting, but dry eye and respiratory signs would not be a feature. Furthermore, the history lacks a stereotypical eating pattern of bulimia nervosa.

(Option D) Parotid gland enlargement and dry mouth due to decreased salivation are seen in mumps, but this patient’s prolonged course of the disease makes this diagnosis very unlikely.

(Option E) Uveoparotid fever or Heerfordt’s syndrome is a rare manifestation of sarcoidosis with the following presentation: